Monday 6 January 2020

MISSION UPSC CURRENT AFF TEST PAPER DATE 29 DEC. 2019 TO 05 JAN. 2020 WITH EXPLACATION


PK PRAJAPAT
Image result for UPSC IMAGES CURRENT AFF  


CURRENT AFF TEST PAPER
DATE 29 DEC. 2019 TO 05 JAN. 2020
WITH EXPLACATION


1.     Consider the following statements regarding NEFT (National Electronic Funds Transfer).
1.      NEFT has become a 24×7 facility on all days except on bank holidays.
2.      NEFT has no upper limit to the transaction amount.
3.      NEFT transactions will only be credited to the beneficiary account in batches, unlike the Immediate Payment Service (IMPS) system.

Which of the above statements is/are correct?

a)     1, 2
b)     1, 3
c)      2, 3
d)     1, 2, 3


2.     Consider the following statements regarding IMPS (Immediate Payment Service).

1.      IMPS is designed to be used for transferring small amounts with caps at 2 lakh per day.
2.      IMPS transactions are not chargeable.
3.      IMPS transaction can be done offline by visiting a bank branch.

Which of the above statements is/are correct?

a)     1 only
b)     1, 2
c)      1, 3
d)     1, 2, 3


3. Consider the following statements regarding Fugitive Economic Offender (FEO).

1.      Fugitive Economic Offender (FEO) is any individual against whom a warrant for arrest in relation to a scheduled offence has been issued by any court in India and the person has left India so as to avoid criminal prosecution.
2.      The special court can declare suo moto an individual as fugitive economic offender if he/she holds properties or the value of such properties believed to be the proceeds of crime.

Which of the above statements is/are correct?

a)     1 only
b)     2 only
c)      Both
d)     None

Image result for UPSC IMAGES CURRENT AFF
4. Consider the following statements with reference to Asiatic Lion.

1.      For the first time, the entire genome of Asiatic lion has been sequenced by scientists from CSIR-Centre for Cellular and Molecular Biology.
2.      At present the only home of Asiatic lion is Gir National Park and Wildlife Sanctuary with over 1000 animals present in the Gir forests.

Which of the above statements is/are correct?

 a) 1 only
 b) 2 only
 c) Both
 d) None

5. Consider the following statements regarding FrogPhone.

1.      It is the world’s first solar-powered remote survey device that will allow scientists to monitor frogs in the wild.
2.      It can be installed at any frog pond and receives a 3G or 4G cellular network.
3.      Environmental data such as air temperature and water temperature can be retrieved through FrogPhone.

Which of the above statements is/are correct?

a)     1, 2
b)     1, 3
c)      2, 3
d)     1, 2, 3


6. White Island, recently seen in news is located in

a)     Australia
b)     New Zealand
c)      Vietnam
d)     Indonesia

7. Consider the following statements regarding Payments Bank.

1.      Payments Bank can accept deposits only upto 2 lakh per individual customer.
2.      Issues debit cards, but not credit cards.
3.      Can distribute financial products such as mutual funds and insurance.

Which of the above statements is/are correct?

a)     1, 2
b)     1, 3
c)      2, 3
d)     1, 2, 3

8. Consider the following statements regarding Small Finance Bank (SFBs).

1.      Small Finance Bank mainly finance small business units, small and marginal farmers and unorganised sector entities.
2.      They do not distribute financial products such as mutual funds and insurance.
3.      Payments banks can apply for conversion into small finance banks SFBs after five years of operation.

Which of the above statements is/are correct?

 a) 1, 2
 b) 2, 3
 c) 1, 3
 d) 1, 2, 3


9.Consider the following statements.

1.      It is a strait between the Persian Gulf and the Gulf of Oman.
2.      It provides the only sea passage from the Persian Gulf to the open ocean.
3.      Almost 25% of total global oil production passes through the strait.

The above statements refer to

a)     Strait of Bab-el-Mandeb
b)     Dardenleez Strait
c)      Strait of Hormuz
d)     Gibraltar Strait

10. In the National Policy for Farmers, 2007 the term ‘FARMER’ includes

1.      Agricultural operational holders
2.      Gardeners
3.      Pastoralists
4.      Person engaged in shifting cultivation
5.      Non-corporate planters

Select the correct answer code:

 a) All except 2
 b) All except 3
 c) All except 5
 d) All of the above

11. Consider the following statements regarding Asian Elephant Specialist Group (AsESG).

1.      The Asian Elephant Specialist Group (AsESG) is an integral part of the Species Survival Commission (SSC) of the international Union for Conservation of Nature (IUCN).
2.      The AsESG provides scientifically-grounded evidence of the abundance, distributions, and demographic status of Asian elephant populations.
3.      Gajah is the journal of the IUCN.

Which of the above statements is/are correct?

 a) 1, 2
 b) 2, 3
 c) 1, 3
 d) 1, 2, 3


12. Consider the following statements regarding Neutrino Observatory (INO) Project.

1.      India-based Neutrino Observatory (INO) is a particle physics research project under construction to primarily study atmospheric neutrinos in a deep cave under INO Peak near Theni in Tamil Nadu.
2.      It is a mega-science project fully funded by the Department of Atomic Energy (DAE).

Which of the above statements is/are correct?

 a) 1 only
 b) 2 only
 c) Both
 d) None

13. Consider the following statements regarding Neutrinos.

1.      Neutrinos are the most widely occurring particle in the universe.
2.      Neutrinos hold the key to several important and fundamental questions on the origin of the Universe and the energy production in stars.
3.      They help in detailed investigation of the structure of the Earth from core onwards.

Which of the above statements is/are correct?

 a) 1, 2
 b) 2, 3
 c) 1, 3
 d) 1, 2, 3

14. Serengeti National Park is located in

a)     Tanzania
b)     Kenya
c)      South Africa
d)     Somalia


15. Consider the following statements.

1.      Recently External Affairs Ministry hosted the Indian Ocean Dialogue and Delhi Dialogue XI.
2.      For the first time, Indian Ocean Dialogue and Delhi Dialogue XI was held consecutively and on similar Indo-Pacific themes.

Which of the above statements is/are correct?

 a) 1 only
 b) 2 only
 c) Both
 d) None

16. Consider the following statements regarding Arctic Council.

1.      The Arctic Council is the leading intergovernmental forum promoting cooperation, coordination and interaction among the Arctic States on issues of sustainable development and environmental protection in the Arctic.
2.      Geneva Declaration lists the Members of the Arctic Council.
3.      Decisions of the Arctic Council are taken by consensus among the Arctic Council States.

Which of the above statements is/are correct?

a)     1, 2
b)     2, 3
c)      1, 3
d)     1, 2, 3

17. Consider the following statements regarding Special Economic Zones (SEZs).

1.      Special Economic Zones (SEZs) are geographically delineated ‘enclaves’ in which regulations and practices related to business and trade differ from the rest of the country.
2.      The SEZ Act 2005 envisages key role for the State Governments in Export Promotion and creation of related infrastructure.
3.      Prior to their introduction, India relied on export processing zones (EPZs) which failed to make an impact on foreign investors.

Which of the above statements is/are correct?

 a) 1, 2
 b) 2, 3
 c) 1, 3
 d) 1, 2, 3
 Image result for UPSC IMAGES CURRENT AFF
18. Which of the following are the incentives and facilities offered to the units in Special Economic Zones (SEZs) in India.

1.      Duty free import/domestic procurement of goods for development, operation and maintenance of SEZ units.
2.      Exemption from Minimum Alternate Tax (MAT).
3.      Single window clearance for Central and State level approvals.

Which of the above statements is/are correct?

 a) 3 only
 b) 1, 2
 c) 1, 3
 d) 2, 3

19. Reciprocal trade agreements (RTAs) include:

1.      Preferential arrangements
2.      Free trade agreements
3.      Customs unions
4.      Common markets

Which of the above statements is/are correct?

 a) 1, 2, 3
 b) 1, 2, 4
 c) 2, 3, 4
 d) 1, 2, 3, 4

20. Consider the following statements regarding Trakea, recently seen in news

1.      It is a unique barcoding software to ensure that forensic reports are not tampered.
2.      Karnataka police force is the first to introduce it in the country.

Which of the above statements is/are correct?

 a) 1 only
 b) 2 only
 c) Both
 d) None


21. Consider the following statements regarding Voice Over Wi-Fi (VoWiFi).

1.      The VoWiFi technology allows users to make outgoing calls to any mobile phone number by connecting to a public WiFi hotspot or any private home WiFi network.
2.      Using VoWiFi service users cannot make calls to a landline number.
3.      All 4G compatible smartphones supports VoWiFi.

Which of the above statements is/are incorrect?

 a) 1, 2
 b) 3 only
 c) 2, 3
 d) 1, 3

22. Consider the following statements regarding Kaleshwaram project.

1.      The Kaleshwaram project is an off-shoot of the original Pranahitha-Chevella Lift Irrigation Scheme.
2.      It is on the Krishna River.
3.      The Centre under the Andhra Pradesh Reorganisation Act, 2014 is mandated to support programmes for the development of backward areas in the successor States,including expansion of physical and social infrastructure.

Which of the above statements is/are correct?

a)     1, 2
b)     2, 3
c)      1, 3
d)     1, 2, 3

23. Consider the following statements regarding Operation Twist.
1.      Operation Twist is a move in which a central bank decides to simultaneously buy long-dated securities while selling short-term securities.
2.      It is the usual practice carried out by RBI for the management of the yield curve.

Which of the above statements is/are correct?

 a) 1 only
 b) 2 only
 c) Both
 d) None

24. Which of the following are the factors in the determination of Minimum Support Prices (MSP).

1.      Cost of production (CoP)
2.      Price trend in the domestic and international markets
3.      Inter-crop price parity

Select the correct answer code:

 a) 1, 2
 b) 1, 3
 c) 2, 3
 d) 1, 2, 3

25. Which of the following is not one of the three important components of Accessible India Campaign.

a)     Built Environment Accessibility
b)     Transportation System Accessibility
c)      Skills and Employment Accessibility
d)     Information and Communication Eco-System Accessibility


26. Consider the following statements regarding National Economic Census.

1.      The national economic census is conducted once in every ten years and is crucial for framing of policies and planning for the government and other organisations.
2.      Recently seventh Economic Census (7th EC) of the Ministry of Statistics and Programme Implementation (MoSPI) was launched.
3.      For the first time, the entire census is being conducted on a digital platform by the use of an application.

Which of the above statements is/are correct?

a)     1, 2
b)     1, 3
c)      2, 3
d)     1, 2, 3


27. Consider the following statements regarding Future Skills initiative.

1.      It is a new age platform built to bridge the industry-academia skill gap and help students keep pace with the emerging technologies.
2.      It is launched by Ministry of Skill Development And Entrepreneurship.

Which of the above statements is/are correct?

a)     1 only
b)     2 only
c)      Both
d)     None

28. Consider the following statements regarding Chabahar Port.

1.      Chabahar Port in located in South-Eastern Iran, on the Gulf of Bab-el-Mandeb.
2.      The Chabahar Port commenced its operations in 2018.
3.      It serves as Iran’s only oceanic port.

Which of the above statements is/are correct?

a)     1, 2
b)     1, 3
c)      2, 3
d)     3 only

29. Consider the following statements.

1.      Supreme Court has declared Right to decent environment including pollution free water as part of Article 21.
2.      Parliament may by law provide for the adjudication of any dispute with respect to the use, distribution and control of waters of any inter-state river and river valley.
3.      Development of inter-state rivers, water supplies, irrigation and canals come under the Union List.

Which of the above statements is/are correct?

a)     1, 3
b)     2, 3
c)      1, 2
d)     1, 2, 3

30. Strand Hogg, recently seen in news is

a)     Earth Resource Satellite
b)     Chicken pox virus
c)      Dangerous Android vulnerability
d)     Long range drone

31. ‘Somalia’ is not bordered by

a)     Eritrea
b)     Djibouti
c)      Ethiopia
d)     Kenya
 Image result for UPSC IMAGES CURRENT AFF
32. Consider the following statements with respect to ‘Central Reserve Police Force (CRPF)’

1.      It was originally constituted as the Crown Representative Police in 1939.
2.      The Parliament Duty Group is an elite CRPF unit tasked with providing armed protection to Parliament House.

Select the correct statements

a)     1 Only
b)     2 Only
c)      Both
d)     None

33. Consider the following statements with respect to ‘Organisation of Islamic Cooperation (OIC)’

1.      India has an observer status at the OIC.
2.      Only African and Asian countries are members of the OIC.

Select the correct statements

 a) 1 Only
 b) 2 Only
 c) Both
 d) None


34. Drake Passage connects

 a) Pacific Ocean and Atlantic Ocean
 b) Arctic Ocean and Atlantic Ocean
 c) Pacific Ocean and Indian Ocean
 d) Arctic Ocean and Pacific Ocean


35. ‘Avangard missile’ is developed by

 a) USA
 b) Russia
 c) Israel
 d) North Korea

36. ‘Mahajan Commission’ is associated with

 a) Karnataka Maharashtra border dispute
 b) Reorganisation of Andhra Pradesh
 c) Inner Line Permit (ILP)
 d) Restricted Area Permit (RAP)

37. The Andaman and Nicobar Islands were under the occupation of which of the following countries during World War II?

 a) Japan
 b) USA
 c) Great Britain
 d) France

38. ‘Ashtadhyayi’ is associated with

 a) Grammar
 b) Mathematics
 c) Astronomy
 d) Chemistry

39. ‘Manasollasa’ is associated with

 a) Someshvara III
 b) Jahangir
 c) Akbar
 d) Tipu Sultan

40. ‘Kalak Tartar’ is located in

 a) Ladakh
 b) Chhota Nagpur
 c) Malwa
 d) Rann of Kutch

41. ‘Silent Valley National Park’ is located in

 a) Kerala
 b) Tamil Nadu
 c) Himachal Pradesh
 d) Arunachal Pradesh


42. Which of the following statements is/are correct with respect to ‘India State of Forest Report 2019’?

While the overall green cover has increased in the country, the forest cover in the north-east has decreased.
North-eastern and central India are the most vulnerable regions to forest fires in India.

Select the correct statements

 a) 1 Only
 b) 2 Only
 c) Both
 d) None

43. Consider the following statements with respect to ‘Protocooperation’

1.      It is an interaction between organisms of different species in which both organisms’ benefit, but neither is dependent on the relationship.
2.      The flowers of plants that are pollinated by insects and birds is an example of protocooperation.

Select the correct statements

 a) 1 Only
 b) 2 Only
 c) Both
 d) None

44. ‘Talley Valley Wildlife Sanctuary’ is located in

 a) Nagaland
 b) Tripura
 c) Ladakh
 d) Arunachal Pradesh

45. Consider the following statements with respect to ‘Solar and Heliospheric Observatory (SOHO)’

1.      It is a joint project of international cooperation between the European Space Agency (ESA) and ISRO.
2.      It was launched onboard Chandrayaan- 2.

Select the correct statements

 a) 1 Only
 b) 2 Only
 c) Both
 d) None

46.‘Line of Actual Control (LAC)’ is concerned with which of the following countries?

 a) China
 b) Myanmar
 c) Pakistan
 d) Bangladesh

47.Consider the following statements with respect to ‘Gaganyaan’

1.      It is an Indian crewed orbital spacecraft that is intended to send 3 astronauts to space for a minimum of seven days by 2022, as part of the Indian Human Spaceflight Programme.
2.      ISRO’s Geosynchronous Satellite Launch Vehicle GSLV Mk III, the three-stage heavy-lift launch vehicle, will be used to launch Gaganyaan.

Select the correct statements

 a) 1 Only
 b) 2 Only
 c) Both
 d) None

48. ‘Long March 5’ heavy-lift rocket is developed by

a)     USA
b)     Russia
c)      China
d)     India

49. Consider the following statements with respect to ‘MANI’ app

1.      It was launched to aid visually impaired persons in identifying denomination of currency notes.
2.      It also authenticates a note as either genuine or counterfeit.

Select the correct statements

 a) 1 Only
 b) 2 Only
 c) Both
 d) None


50. ‘SnowEx’ is snow remote sensing programme launched by

 a) NASA
 b) ISRO
 c) ESA
 d) Roscosmos


51. The northeast monsoon generally brings rain to

1.      Tamil Nadu 
2.      Puducherry
3.      Kerala

Select the correct code:

 a) 1 and 2
 b) 2 and 3
 c) 1 and 3
 d) All of the above

52. ‘Palau’ is located in

 a) Pacific Ocean
 b) Indian Ocean
 c) Atlantic Ocean
 d) Arctic Ocean


53. Consider the following statements with respect to ‘GEMINI’

1.      It aims to disseminate emergency information and communication on disaster warnings, Potential Fishing Zones (PFZ) and Ocean States Forecasts (OSF) to fishermen.
2.      It was launched by Indian National Centre for Ocean Information Services (INCOIS) and Airport Authority of India (AAI).

Select the correct statements

 a) 1 Only
 b) 2 Only
 c) Both
 d) None


54. Which of the following pairs is/are correctly matched?

1.      Cyclone ‘PABUK’ – Northeast and adjoining east central Arabian sea
2.      Cyclone ‘FANI’ – Southwest Bay & adjoining Southeast of Bengal
3.      Cyclone ‘HIKAA’ – Andaman Sea

Select the correct code:

 a) 1 and 2
 b) 2 Only
 c) 1 and 3
 d) 1, 2 and 3

55. Consider the following statements with respect to ‘Nageswaraswamy Temple’.

1.      It is an ancient Chola temple, constructed by Aditya Chola.
2.      It was built in the Kalinga region.

Select the correct statements

 a) 1 Only
 b) 2 Only
 c) Both
 d) None

56. Which of the following statements is/are correct?

1.      The Eat Right India Movement is an initiative of the Ministry of Women and Child Development.
2.      It involves providing fortified food to infants and lactating mothers at Anganwadi Centres.

Options:

a. 1 only
b. 2 only
c. Both 1 and 2
d. Neither 1 nor 2


57. The term Miyawaki is associated with which of the following?

a)     Traditional dance of Tibet
b)     Afforestation method
c)      A musical instrument from Arunachal Pradesh
d)     Traditional silk weaving technique

58. Which of the following statements is/are correct?

1.      The Pradhan Mantri Jan Arogya Yojana provides an insurance cover of Rs.5 lakh per individual.
2.      The PMJAY beneficiaries get an e-card that can be used to avail services at any hospital, public or private, anywhere in the country.
3.      The eligibility for the PMJAY is based on the SECC 2011 data.

Options:

a. 1,2 and 3
b. 1 and 3
c. 2 and 3
d. 3 only

Image result for UPSC IMAGES CURRENT AFF
59. Arrange the following cities from North to South:

1.      Sydney
2.      Perth
3.      Brisbane
4.      Melbourne
5.      Canberra

Options:

a.       3, 2, 1, 5, 4
b.      3, 1, 2, 5, 4
c.       2, 3, 1, 4, 5
d.      2, 3, 4, 1, 5


60. Which of the following states have been accorded special provisions under
 Article 371 of the Indian Constitution:

1.      Sikkim
2.      Karnataka
3.      Tripura
4.      Maharashtra
5.      Mizoram
6.      Nagaland

Options:

a) 1,3,5 and 6
b) 1,5 and 6
c) 1,2,4,5 and 6
d) 1,2,4 and 6


61. Which of the following statements are correct?

1.      The Chairperson of the Rajya Sabha nominates the members of the privileges committee of Rajya Sabha.
2.      The Privileges Committee of Rajya Sabha consists of ten members.

Options:

a) 1 only
b) 2 only
c) Both 1 and 2
d) Neither 1 nor 2

62. which of the following statements are correct?
1.      The six zonal councils in India have been created under the States Re-Organization Act, 1956.
2.      The councils function under the aegis of the Ministry of Home Affairs’ Inter-State Council Secretariat.

Options:

a) 1 only
b) 2 only
c) Both 1 and 2
d) Neither 1 nor 2

63. Arrange the following ports from north to south:

1.      Krishnapatnam port
2.      Vishakapatnam port
3.      New Mangalore port
4.      Mormugao port
5.      Ennore port

Options:

a) 2,4,1,5,3
b) 2,1,4,3,5
c) 2,4,1,3,5
d) 1,2,4,3,5

64. Which of the following statements is/are correct?

1.      The Bureau of Energy Efficiency is an agency of the Government of India, under the Ministry of Petroleum and Natural Gas.
2.      It was created under the provisions of the National Renewable Energy Act.

Options:

a. 1 only
b. 2 only
c. Both 1 and 2
d. Neither 1 nor 2

65. During the Third Anglo-Maratha War, the British East India Company fought against a band  of Muslim mercenaries known as –

a. Pindaris
b. Mappilas
c. Santhals
d. Bhills

66. Who appoints the State Election Commissioner, for overseeing elections to the local bodies?

a. President
b. Ministry of Home Affairs
c. Governor
d. Chief Minister

67. Which of the following statements is/are correct?

1.      The Global Code of Practice on the International Recruitment of Health Personnel was adopted by the WHO’s 63rd World Health Assembly in 2010.
2.      It is the main international policy framework for addressing shortages and maldistribution of healthcare professionals.

Options:

a. 1 only
b. 2 only
c. Both 1 and 2
d. Neither 1 nor 2

68. Which of the following statements is/are correct?
1.      A cryogenic rocket engine uses a cryogenic fuel and/or oxidizer which is liquefied and stored at extremely low temperatures.
2.      The upper stage of GSLV MK-III launch vehicle is powered by Cryogenic Engine (CE)-20.

Options:

a. 1 only
b. 2 only
c. Both 1 and 2
d. Neither 1 nor 2

69. With respect to NISHTHA Programme, consider the following statements

1.      It aims to identify and encourage talented children to enrich their skills and knowledge.
2.      It was launched by the Ministry of Human Resources and Development.

Which of the statement(s) given above is/are correct?

a.  1 only
b.  2 only
c.  Both 1 and 2
d.  Neither 1 nor 2

70. Consider the following statements with respect to Patola Saree

1.      It is a trademark Saree from the state of Assam.
2.      It is considered to be very costly and worn only by the Royals or the Aristocrat.

Which of the statement(s) given above is/are correct?

a.  1 only
b.  2 only
c.  Both 1 and 2
d.  Neither 1 nor 2


71. Consider the following statements

1.      It is also known as the “Khajuraho of Vidarbha”.
2.      It is situated on the bank of River Wainganga.
3.      The temples belong to the Nagara group of temples of North India.

Identify the temple that correctly matches with the above description:

a.  Airavatesvara Temple
b.  Chamunda Devi Temple
c.  Markandeshwar Temple
d.  Padmanabhaswamy Temple


72.Consider the following statements with respect to E-visa

1.      It is an online process where there is no paper work is involved and all the process happens online.
2.      It is valid for entry only through Airports and not seaports.
3.      One can apply e-visa at the airport.

Which of the statement(s) given above is/are correct?

a.  1 only
b.  1 and 2 only
c.  1 and 3 only
d.  1, 2 and 3


73.Performance Grading Index (PGI) aims to evaluate the performance of which of the following?

a.  Cyber Safety
b.  School Education System
c.  Rural and Urban Sanitation
d.  Aspirational Districts Programme

74.Consider the following statements with respect to the World Health Organization (WHO)

1.      World Drug Report is published by WHO.
2.      2020has been designated as the International Year of the Nurse and the Midwife.

Which of the statement(s) given above is/are correct?

a.  1 only
b.  2 only
c.  Both 1 and 2
d.  Neither 1 nor 2


75.EVALIwhich was in news recently is related to?

a.  Vaping-Related Illness
b.  A new Antibiotic resistance Bacteriophage
c.  A newly synthesized Protein
d.  None of the above

76.Consider the following statements with respect to Securities and Exchange Board of India (SEBI)

1.      The Chairman of the SEBI is appointed by the President of India.
2.      It has the same power as vested in a civil court while trying any suit.

Which of the statement(s) given above is/are not correct?

a.  1 only
b.  2 only
c.  Both 1 and 2
d.  Neither 1 nor 2

77. Extraocular Visionsometimes seen in news is related to?

a.  Trouble focusing when reading or looking at a computer
b.  The Blurred vision of the objects at a distance
c.  The ability to see without eyes
d.  Condition where the optic nerve of the eye is affected


78. A First-of-its-kind Rehabilitation centre for Freshwater Turtles will be inaugurated in?

a.  Odisha
b.  West Bengal
c.  Bihar
d.  Assam


79.Kalapani border issue sometimes seen in the news recently is related to which of the following two countries?

a.  India and Nepal
b.  India and Bhutan
c.  India and Bangladesh
d.  India and Afghanistan

80. With respect to Sugauli Treaty of 1816, consider the following statements

1.      It was signed between the British East India Company and the Kingdom of Gorkha.
2.      The treaty was signed on the conclusion of the Anglo-Nepalese War and it ceded some Nepalese controlled territory to the British.

Which of the statement(s) given above is/are correct?

a.  1 only
b.  2 only
c.  Both 1 and 2
d.  Neither 1 nor 2

81. Consider the following statements with respect to Savitribai Phule

1.      She was the first woman Governor of an Indian state after independence.
2.      She led the Dharasana Satyagraha with other leaders in 1930.
3.      She started Mahila Seva Mandal in 1852, which worked for raising women’s consciousness about their human rights, dignity of life and other social issues.
4.      She started the first ever infanticide prohibition home of India in 1853.

Which of the statement(s) given above is/are correct?

a.  1 and 2 only
b.  1 and 3 only
c.  2 and 4 only
d.  3 and 4 only

82. Cyber Safe Women’ initiative was launched recently by which of the following states?

a.  Assam
b.  Kerala
c.  Maharashtra
d.  Andhra Pradesh

83. Lai Haraoba is the ritual festival celebrated by which of the following communities?

a.  Manipuri meiteis
b.  Deori of Assam
c.  Agariya of Chhattisgarh
d.  Bakarwal of Jammu and Kashmir


84. Consider the following statements with respect to the Stock Markets in India

Bear market refers to a market condition in which the prices of securities are falling or are expected to fall.
BSE (Bombay Stock Exchange) and NSE (National Stock Exchange) are the only two Stock Exchanges functioning in India.

Which of the statement(s) given above is/are correct?

a.  1 only
b.  2 only
c.  Both 1 and 2
d.  Neither 1 nor 2


85. Consider the following statements with respect to Gaganyaan Mission

India will become the 3rd country after Russia and USA to launch human spaceflight mission.
Gaganyaan is an indigenous mission that would take Indian astronauts to space.
Which of the statement(s) given above is/are correct?

a.  1 only
b.  2 only
c.  Both 1 and 2
d.  Neither 1 nor 2


86. Consider the following statements with respect to Monsoon

When the ITCZ is shifted to north of the Equator, the southeast trade wind changes to a southwest wind as it crosses the Equator.
The Indian climate is characterized by the complete reversal of wind system with the change of season in a year.

Which of the statement(s) given above is/are correct?

a.  1 only
b.  2 only
c.  Both 1 and 2
d.  Neither 1 nor 2

87. MANI app which was in news is related to,

a.  Real Time trading of Cryptocurrencies
b.  UPI based instant real-time payment system
c.  To help visually-impaired people to identify the denomination of currency notes
d.  None of the above

88. Consider the following

1.      Iraq
2.      Turkmenistan
3.      Kuwait
4.      Armenia
5.      Syria

Which of the countries given above do not share border with Iran?

a.  3 and 4 only
b.  2 and 5 only
c.  1 and 4 only
d.  3 and 5 only


89. With reference to the new wagon “BLCS (type A & B)”, consider the following statements:

1. They are manufactured at the Golden Rock Railway Workshop in Tiruchirappalli, Tamil Nadu.
2. It will help boost the cargo carrying capacity by four times due to increased length of the train, use of double-stack containers and more payload carrying capability.

Which of the statements given above is/are correct?

A.         1 only
B.         2 only
C.         Both 1 and 2
D.         Neither 1 nor 2

90. With reference to the 2019 report released by Forest Survey of India (FSI) on Forest Fires, consider the following statements:

1. About 21.4% of forest cover in India is prone to fires, with forests in the south India being the most vulnerable.
2. One of the major reasons for forest fires in the north-east is slash-and-burn cultivation, commonly called jhoom or jhum cultivation.

Which of the statements given above is/are correct?

A.         1 only
B.         2 only
C.         Both 1 and 2
D.         Neither 1 nor 2

91. With reference to the Indian Railway Protection Force Service, consider the following statements:

1. Recently the Railways renamed the Railway Protection Force (RPF) as Indian Railway Protection Force Service and accorded it organised Group A status (OGAS).
2. Railway Protection Force (RPF) is responsible for all aspects of railways security.

Which of the statements given above is/are correct?

A.         1 only
B.         2 only
C.         Both 1 and 2
D.         Neither 1 nor 2

92. With reference to the Dornier 228 aircraft, consider the following statements:

1.It is a multi-purpose, fuel efficient, rugged, light weight twin turboprop aircraft with a retractable tricycle landing gear.
2.The modified Dornier 228 aircraft have been acquired to undertake in-house calibration of Navigational aids available after implementation of Modernised Airfield Infrastructure (MAFI) at IAF bases.

Which of the statements given above is/are correct?

            A.         1 only
            B.         2 only
            C.         Both 1 and 2
            D.         Neither 1 nor 2

93. The Belum Caves, often mentioned in news, are located in:

A.         Andhra Pradesh
B.         Karnataka
C.         Kerala
D.         Tamil Nadu

94. Amjad Ali Khan is an/a:

A. Indian classical sarod player
B. Indian tabla maestro
C. Celebrated painter of Vellore during British rule
D.         One of the greatest Carnatic music vocalists

95. With reference to the Vistadome Coach, consider the following statements:
1. It consists of a glass roof which has electrically controlled opalescence.
2. Him Darshan Express will be the first-ever train with Vistadome coaches by the Indian Railways that will run on a regular basis.

Which of the statements given above is/are correct?

A.         1 only
B.         2 only
C.         Both 1 and 2
D.         Neither 1 nor 2

96. With reference to the EVALI (e-cigarette, or vaping, product use associated lung injury), consider the following statements:
1. It is the name given by the World Health Organization (WHO) to the dangerous, newly identified lung disease linked to vaping.
2. In the most severe and life-threatening cases, it causes the lungs to stop functioning altogether.

Which of the statements given above is/are correct?

A.         1 only
B.         2 only
C.         Both 1 and 2
D.         Neither 1 nor 2

97. Travel and Tourism Competitiveness Index is released by which of the following?

A.         World Economic Forum
B.         NITI Aayog
C.         World Bank
D.         Amnesty International

98. With reference to the ‘World Braille Day’, consider the following statements:

1. The day is marked to remember the birth anniversary of Louis Braille, the inventor of Braille - for people with visual disabilities.
2. The Braille Script consists of raised dots arranged in “cells”.

Which of the statements given above is/are correct?

A.         1 only
B.         2 only
C.         Both 1 and 2
D.         Neither 1 nor 2

99. With reference to Islamic Cooperation countries (OIC), consider the following statements

1. It is the second largest inter-governmental
organization after the United Nations.
2. The OIC has permanent delegations to the
United Nations and the European Union.
3. India is one of its founding member

Choose the correct option from the following

(a) 1 and 2 only
(b) 2 and 3 only
(c) 1 and 3 only
(d) 1, 2 and 3

100.  “Nagpur Resolution”, which was seen
recently in news, refers to?

(a) It is United Nations Security Council
resolution that extends the sanctions
against the Central African Republic.
(b) The Convention establishes rules of
airspace, aircraft registration and safety,
security, and sustainability, and details
the rights of the signatories in relation
to air travel.
(c) It is a convention that plays a role in the
implementation of several international
conventions on nature conservation and
biodiversity.
(d) It is to empower the citizens by policy
interventions for better service delivery
through timely updation of citizen’s
charters, implementation of enactments
and benchmarking standards for
continuous improvement.

101. With reference to Khadi and Village Industries Commission (KVIC), consider the following statements

1. It is a statutory body.
2. It is an apex organization under the Ministry of tribal affairs.
3. It has the economic objective of producing saleable articles.

Choose the correct option from the following
(a) 1 and 2 only
(b) 2 and 3 only
(c) 1 and 3 only
(d) 1, 2 and 3

102. With reference to Good Governance
Index, consider the following statements

1. It was launched by NITI Aayog.
2. Tamil Nadu topped the Good Governance
Index.

Choose the correct option from the following
(a) 1 only
(b) 2 only
(c) Both 1 and 2
(d) Neither 1 nor 2

103. With reference to Rohtang Tunnel,
consider the following statements

1. It has been recently named after VD savarkar
2. Upon completion, it will be the world’s
longest highway tunnel at an altitude of
above 10,000 feet.
3. It will reduce the distance between North Jammu and North Kashmir.

Choose the correct option from the following
(a) 1 and 2 only
(b) 2 only
(c) 3 only
(d) 1, 2 and 3

104. With reference to Atal Bhujal Yojana,
consider the following statements

1. It is a scheme for Rainwater harvesting.
2. It is partly funded by World Bank.
3. Implementation of the scheme would be pan India.

Choose the correct option from the following

(a) 1 and 2 only
(b) 2 only
(c) 3 only
(d) 1, 2 and 3

105. Consider the following statements regarding NPR - National Population Register

1. The database will have demographic
details.
2. NPR is to create a comprehensive identity
database of every “usual resident” of the
country.
3. The process of updating NPR will be carried
out under the aegis of the Registrar General
and ex-Offi cio Census Commissioner.

Choose the correct option from the following

(a) 1 and 2 only
(b) 2 and 3 only
(c) 1 and 3 only
(d) 1, 2 and 3

106.Consider the following statements regarding Joint Comprehensive Plan of Action (JCPOA)
1. It is also known as the “Iran deal
2. The agreement is made between Iran, the
P5+1 and the European Union.

Choose the correct option from the Following

(a) 1 only
(b) 2 only
(c) Both 1 and 2
(d) Neither 1 nor 2

107. With reference to Ebola Virus Disease
(EVD), consider the following statements

1. It was first discovered in Uganda.
2. People can get EVD through direct contact
with an infected animal (bat or nonhuman
primate) or a sick or dead person infected
with Ebola virus.

Choose the correct option from the following

(a) 1 only
(b) 2 only
(c) Both 1 and 2
(d) Neither 1 nor 2

108. Consider the following facts about Sambhar lake:

1. The Sambhar Salt Lake is India’s largest inland saltwater body located near Jaipur in Rajasthan.
2. Sambhar has not been designated as a Ramsar site.

Which of the above statement(s) is/are correct?

(a) 1 only
(b) 2 only
(c) Both 1 and 2
(d) Neither 1 nor 2

109. Which of the following are the objectives
of Bharat Darshan Scheme?

1. To position tourism as a major engine of
economic growth and job creation;
2. Develop circuits having tourist potential in
a planned and prioritized manner
3. Promote cultural and heritage value of
the country to generate livelihoods in the
identifi ed regions
4. Build rural infrastructure

Choose the correct option

(a) 1, 2 and 3
(b) 1 and 2
(c) 1, 2, 3 and 4
(d) 1 and 3

110. National Green Corps ‘Eco Club’
Programme that was in news recently
aims at

(a) To provide opportunities for children
to understand the environment and
environmental problems through school
eco-clubs.
(b) To make polluting industries conscious of
the importance of the environment for
the existence of life.
(c) To conserve tiger population
(d) To protect the endangered wildlife

111. Consider the following statements about National Tiger Conservation Authority (NTCA)
1. Project Tiger was launched in 1973 with 9
tiger reserves for conserving our national
animal, the tiger.
2. The National Tiger Conservation Authority
(NTCA) is a statutory body of the
Ministry, with an overarching supervisory/
coordination role, performing functions as
provided in the Wildlife (Protection) Act,
1972.

Choose the correct option

(a) 1 only
(b) 2 only
(c) Both 1 and 2
(d) Neither 1 nor 2

112. Quick Reaction Surface to Air Missile
(QRSAM) system was successfully test fired by DRDO recently. Which of the
following statements are incorrect about Quick Reaction Surface to Air Missile (QRSAM) system?

1. It uses solid fuel propellant and has a strike range of 25-30 km with capability of hitting multiple targets.
2. The missile is an all-weather, allterrain surface-to-air missile equipped with electronic counter measures against jamming by aircraft radars
3. It will be commissioned to Indian Navy.

Choose the correct option

(a) Only 1
(b) 1 and 2 only
(c) 1, 2 and 3
(d) Only 3

113. Which of the following committee recommended the institution of collegium system for the appointment of chief election commissioner and election commissioners?

(a) Dinesh Goswami Committee
(b) Administrative Reforms Committee
(c) Punchhi Commission
(d) Sarkaria Commission

114. Match the following pairs correctly
 Basin Type    Explanation
1. Category-   I A. Basins with
 established
 commercial
 production.
2. Category-   II B. Basins with known
 accumulation of
 hydrocarbons but
 no commercial
 production achieved
 so far
3. Category-   III C. Basins having
 uncertain potential
 which may be
 prospective by
 analogy with similar
 basins in the world.
4. Category-   IV D. Basins having
 hydrocarbon shows
 that are considered
 geologically
 prospective

Choose the correct option

(a) 1-A, 2-B, 3-D, 4-C
(b) 1-A, 2-B, 3-C, 4-D
(c) 1-D, 2-C, 3-A, 4-B
(d) 1-C, 2-A, 3-D, 4-B

115. What are the various problems that currently ail Indian oil and gas sector?

1. Declining domestic crude production
2. Large crude import bills
3. Inadequate transmission & distribution infrastructure
4. Large share of MNCs

Choose the correct option

(a) 1, 2 and 3
(b) 1 and 2
(c) 1, 2, 3 and 4
(d) 3 and 4

116. Consider the following statements regarding Quick Reaction Surface to Air Missile (QRSAM) system,

1. It is developed by Defence Research and Development Organization (DRDO).
2. It failed fl ight test when conducted from Integrated Test Range, Chandipur.
3. The system is compact with minimum number of vehicles for a fi ring unit.

Which of the following statements is/are correct?

(a) 1 and 2 only
(b) 2 only
(c) 1 and 3 only
(d) 1, 2 and 3

117. Consider the following statements
regarding Chief of Defense Staff,

1. It will be in the rank of four star general.
2. The Chief of Defence Staff will head the Department of Military Affairs (DMA).
3. One of the task of CDS will be Promoting use of indigenous equipment by the Services.

Which of the following statements is/are correct?

(a) 1 and 2 only
(b) 2 only
(c) 1 and 3 only
(d) 1, 2 and 3

118. Consider the following statements regarding Atal Bhujal Yojana (ATAL JAL),
1. It is a Central Sector Scheme.
2. It will be implemented over a period of 5
years (2020-21 to 2024-25)
3. The scheme aims to improve ground water management.

Which of the following statements is/are correct?

(a) 1 and 2 only
(b) 2 only
(c) 1 and 3 only
(d) 1, 2 and 3

119. Consider the following statements
regarding Indian Railway Management
Service (IRMS),

1.It has unifi ed the existing eight Group A services of the Railways into a Central Service.
2.Unification of ser vices had been recommended by various committees for reforming Railways including - the Prakash Tandon Committee (1994)
3.Decisions regarding IRMS have been taken by the Railway Ministry.

Which of the following statements is/are correct?

(a) 1 and 2 only
(b) 2 only
(c) 1 and 3 only
(d) 1, 2 and 3

120. Consider the following statements
regarding Census,

1.Decennial Population Census is being conducted in India synchronously since 1872 without break.
2.Census 2021 will be 16th Census in the country and 8th after independence.

Which of the following statement is/are correct?

(a) 1 only
(b) 2 only
(c) Both 1 and 2
(d) Neither 1 nor 2

121. Consider the following statements regarding recently unveiled India’s First CNG Bus which can run 1000 Kms in one fill,

1.      The project has been executed by Indraprastha Gas Limited (IGL).
2.      It has been has been achieved through  pioneering design of Type IV Composite Cylinders in buses, replacing traditional very heavy Type-I Carbon Steel cylinders.

Which of the following statement is/are correct?

(a) 1 only
(b) 2 only
(c) Both 1 and 2
(d) Neither 1 nor 2

122. Consider the following statements regarding SDG India Index,

1.      2019 will be the second edition of the Sustainable Development Goals (SDG) India Index.
2.      The index documents the progress made by India’s States and Union Territories towards implementing the 2030 SDG targets.
3.      It has been developed in collaboration with the Ministry of Statistics and Programme Implementation (MoSPI), the United Nations in India, and the Global Green Growth Institute.

Which of the following statements is/are correct?

(a) 1 and 2 only
(b) 2 only
(c) 1 and 3 only
(d) 1, 2 and 3

123.. Consider the following statements regarding Good Governance Index,

1.It is used to provide quantifi able data to compare the state of governance in all states and UTs.
2.The index does not consider Public Health and Infrastructure.
3.All the indicators are given same weightage under one Governance Sector to calculate the value.

Which of the following statements is/are correct?

(a) 2 and 3 only
(b) 1 only
(c) 3 only
(d) 1, 2 and 3

JATF LOGO-1.jpg
29  DEC 2019 TO 05 JAN 2020 CURRENT AFF
ANSWER KEY


1.     Solution: c)
Starting December 16, you can now transfer money online, using the National Electronic Fund Transfer (NEFT) system at any time of the day, all through the year. Earlier, NEFT transactions could only be carried out between 8 am to 7 pm on weekdays plus first and third Saturday — second and fourth Saturdays, plus Sundays, excluded.
Moreover, NEFT transactions weren’t available on public holidays — which is not the case now.
NEFT has no upper limit to the transaction amount. This means that you can now transfer large amounts round-the-clock through NEFT. “This will be extremely beneficial for large-value transactions which cannot be done through UPI or IMPS
NEFT transactions will only be credited to the beneficiary account in batches, unlike the Immediate Payment Service (IMPS) system, which transfers money in real time.

2. Solution: a)
RBI announced that processing charges levied for NEFT and RTGS transactions will not apply starting January 2020. IMPS, on the other hand, is still chargeable. While the charge levied on an IMPS transaction depends largely on the amount being transferred and the policy of the bank carrying out the transfer, the fees usually range from 1 to 25.
Unlike IMPS, which can only be transacted online, a NEFT transaction can be done offline by visiting a bank branch. Additionally, as per the RBI mandate, there’s no limit on the amount of money that may be transferred via NEFT — unlike IMPS, where a maximum of Rs 2 lakh per day can be transferred.

3. Solution: a)
Who is a Fugitive Economic Offender (FEO)?
An FEO is defined by The Fugitive Economic Offenders (FEO) Act, 2018 as “any individual against whom a warrant for arrest in relation to a scheduled offence has been issued by any court in India, who (i) has left India so as to avoid criminal prosecution; or (ii) being abroad, refuses to return to India to face criminal prosecution”.
What is the process for declaring an individual an FEO?
Under the Act, an application must be filed in the special court asking that a particular individual may be declared an FEO.
The application must be accompanied by “reasons for the belief that an individual is a fugitive economic offender; any information available as to the whereabouts of the fugitive economic offender; a list of properties or the value of such properties believed to be the proceeds of crime”, etc.

4. Solution: a)
For the first time, the entire genome of Asiatic lion has been sequenced by scientists from CSIR-Centre for Cellular and Molecular Biology, Hyderabad.
The objective is to understand the species at DNA level and study if there are any specific problems with regard to adaptability to environment or behaviour vis-à-vis other big cats.
About Asiatic Lion:
IUCN Red List Status: Endangered
Listed in Schedule I of Wildlife (Protection) Act 1972, in Appendix I of Convention on International Trade in Endangered Species (CITES).
At present the only home (natural habitat) of Asiatic lion is Gir National Park and Wildlife Sanctuary in Gujarat.
The population of the endangered Asiatic lion is very low — around 523 animals are present in the Gir forests.


5. Solution: d)
Researchers have developed a device that will allow scientists to monitor frogs in the wild. Described as the world’s first solar-powered remote survey device that can be installed at any frog pond and which receives a 3G or 4G cellular network, it has been named “FrogPhone”.
It has been developed by a team from various Australian institutions, including the University of New South Wales and the University of Canberra.
With FrogPhone, researchers can simply “call” a frog habitat. After a call is made to one of the FrogPhones already on a site, the device will take three seconds to receive it. During these few seconds, the device’s temperature sensors will get activated, and environmental data such as air temperature, water temperature and battery voltage will be sent to the caller’s phone via a text message.
Because frogs are most active during night, researchers are usually required to make nightly observations in order to monitor them on site. The FrogPhone will allow researchers to dial these devices remotely, and analyse the data later.
It will reduce costs and risks, including the negative impact of human presence on the field site, the researchers say. These devices also allow for monitoring of local frog populations more frequently than before, which is important because these populations are recognised as indicators of environmental health.


6. Solution: b)
Whakaari /White Island volcano in New Zealand erupts.
7. Solution: c)
Going_Further_Afiled
8. Solution: c)
 RBI has announced the final guidelines for on-tap licencing of private sector SFBs.
These guidelines include:
Payments banks can apply for conversion into small finance banks (SFBs) after five years of operation.
The promoter of a payments bank is eligible to set up an SFB, provided that both banks come under the non-operating financial holding company (NOFHC) structure.
The minimum paid-up capital requirement for SFBs has been raised from  100 crore to 200 crore.
SFBs should be listed within three years of reaching a net worth of  500 crore. They will be given scheduled bank status immediately upon commencement of operations,and will have general permission to open banking outlets from the date of commencement of operations.

9. Solution: c)
 Strait of Hormuz is a strait between the Persian Gulf and the Gulf of Oman. It provides the only sea passage from the Persian Gulf to the open ocean and is one of the world’s most strategically important choke points.

10. Solution: d)
In the National Policy for Farmers, which was officially approved by the Centre in 2007 there is a clear and comprehensive definition available for the term farmer.
‘For the purpose of this Policy, the term ‘FARMER’ is referred to a person actively engaged in the economic and/or livelihood activity of growing crops and producing other primary agricultural commodities.
It will include all agricultural operational holders, cultivators, agricultural labourers, sharecroppers tenants, poultry and livestock rearers, fishers, beekeepers, gardeners, pastoralists, non-corporate planters and planting labourers.
It will also include persons engaged in various farming related occupations such as sericulture, vermiculture and agro-forestry.
The term will also include tribal families / persons engaged in shifting cultivation and in the collection, use and sale of minor and non-timber forest produce.
The National Policy for Farmers was drafted by the National Commission of Farmers headed by M.S. Swaminathan.

11. Solution: d)
The Asian Elephant Specialist Group (AsESG) is an integral part of the Species Survival Commission (SSC) of the International Union for Conservation of Nature (IUCN). The AsESG shall provide the best available scientifically grounded evidence to the abundance, distribution, and demographic status of Asian elephant populations in all 13 range states.
Gajah is the bi-annual journal of the IUCN/SSC Asian Elephant Specialist Group (AsESG).

12. Solution: a)
The Centre has reiterated that the Indian Neutrino Observatory (INO) will be established in picturesque Theni in south Tamil Nadu.
The India-based Neutrino Observatory (INO) Project is a multi-institutional effort aimed at building a world-class underground laboratory with a rock cover of approx.1200 m for non-accelerator based high energy and nuclear physics research in India. The initial goal of INO is to study neutrinos.
It is a mega-science project jointly funded by the Department of Atomic Energy (DAE) and the Department of Science and Technology (DST).
The project includes:
Construction of an underground laboratory and associated surface facilities at Pottipuram in Bodi West hills of Theni District of Tamil Nadu.
Construction of an Iron Calorimeter (ICAL) detector for studying neutrinos.
Setting up of National Centre for High Energy Physics at Madurai, for the operation and maintenance of the underground laboratory, human resource development and detector R&D along with its applications.
13. Solution: b)
Neutrinos, first proposed by Swiss scientist Wolfgang Pauli in 1930, are the second most widely occurring particle in the universe, only second to photons, the particle which makes up light. In fact, neutrinos are so abundant among us that every second, there are more than 100 trillion of them passing right through each of us — we never even notice them.
Neutrinos hold the key to several important and fundamental questions on the origin of the Universe and the energy production in stars. Another important possible application of neutrinos is in the area of neutrino tomograph of the earth, that is detailed investigation of the structure of the Earth from core onwards. This is possible with neutrinos since they are the only particles which can probe the deep interiors of the Earth.
14. Solution: a)
Since 1974, the population of herbivorous in Gir forest has been on the rise. In 2013, the population of ungulates was estimated to be 1,26,893 or 76.49 animals per square kilometres. That translates to 8000 kg of biomass available to carnivorous, very close to the levels in Serengeti National Park in Tanzania.
15. Solution: c)
External Affairs Ministry hosted the 6th Indian Ocean Dialogue and Delhi Dialogue XI at the Pravasi Bharatiya Kendra in New Delhi.
The theme for Indian Ocean Dialogue is “Indo-Pacific: Re-imagining the Indian Ocean through an Expanded Geography”.
The theme for Delhi Dialogue is “Advancing Partnership in Indo-Pacific”, and is being organized with the assistance of the Research and Information System for Developing Countries.

16. Solution: c)
The Arctic Council is the leading intergovernmental forum promoting cooperation, coordination and interaction among the Arctic States, Arctic indigenous communities and other Arctic inhabitants on common Arctic issues, in particular on issues of sustainable development and environmental protection in the Arctic.
The Ottawa Declaration lists the following countries as Members of the Arctic Council: Canada, the Kingdom of Denmark, Finland, Iceland, Norway, the Russian Federation, Sweden and the United States.
Observer status in the Arctic Council is open to non-Arctic states, along with inter-governmental, inter-parliamentary, global, regional and non-governmental organizations that the Council determines can contribute to its work.
Arctic Council assessments and recommendations are the result of analysis and efforts undertaken by the Working Groups. Decisions of the Arctic Council are taken by consensus among the eight Arctic Council States, with full consultation and involvement of the Permanent Participants.
The Chairmanship of the Arctic Council rotates every two years among the Arctic States.

17. Solution: d)
Special Economic Zones (SEZs) are geographically delineated ‘enclaves’ in which regulations and practices related to business and trade differ from the rest of the country and therefore all the units therein enjoy special privileges.
The basic idea of SEZs emerges from the fact that, while it might be very difficult to dramatically improve infrastructure and business environment of the overall economy ‘overnight’, SEZs can be built in a much shorter time, and they can work as efficient enclaves to solve these problems.
SEZs were introduced to India in 2000, following the already successful SEZ model used in China. Prior to their introduction, India relied on export processing zones (EPZs) which failed to make an impact on foreign investors. By 2005, all EPZs had been converted to SEZs.

18. Solution: c)
The incentives and facilities offered to the units in SEZs for attracting investments into the SEZs, including foreign investment include:-
Duty free import/domestic procurement of goods for development, operation and maintenance of SEZ units
100% Income Tax exemption on export income for SEZ units under Section 10AA of the Income Tax Act for first 5 years, 50% for next 5 years thereafter and 50% of the ploughed back export profit for next 5 years. (Sunset Clause for Units will become effective from 01.04.2020)
Exemption from Minimum Alternate Tax (MAT) under section 115JB of the Income Tax Act. (withdrawn w.e.f. 1.4.2012)
Exemption from Central Sales Tax, Exemption from Service Tax and Exemption from State sales tax. These have now subsumed into GST and supplies to SEZs are zero rated under IGST Act, 2017.
Other levies as imposed by the respective State Governments.
Single window clearance for Central and State level approvals.

19. Solution: d)
Countries use bilateral/regional trade agreements to increase market access and expand trade in foreign markets. These agreements are called reciprocal trade agreements (RTAs) because members grant special advantages to each other.
 RTAs include many types of agreements, such as preferential arrangements, free trade agreements, customs unions, and common markets, in which members agree to open their markets to each other’s exports by lowering trade barriers.

20. Solution: a)
Haryana Police has adopted a unique barcoding software — Trakea — to ensure that thousands of forensic reports that form the backbone of the criminal investigation system and subsequent trials in the courts of law, are not tampered with.
According to the police, Trakea ensures foolproof security of the samples collected from the scene of crime, and the forensic analysis reports, and is different from traditional methods that the state police force has been following for decades.
Haryana Police claims it is the country’s first police force to have introduced this unique barcoding for forensic reports.
Trakea is aimed at ensuring security and a tamperproof tracking system for forensic reports. It streamlines the functioning of Forensic Science Laboratories.
Essentially, it is a forensic evidence management system that helps in automation of the entire procedure, right from the stage when forensic experts collect vital samples from the scene of crime to conducting analysis of the samples, followed by tracking casewise forensic reports electronically through barcodes.
Even the selection of forensic teams is done randomly through this software.

21. Solution: c)
Following its introduction in Delhi/NCR, Bharti Airte commercially rolled-out its Voice Over Wi-Fi (VoWiFi) service –‘Airtel Wi-Fi Calling’ in Mumbai, Kolkata, Andhra Pradesh, Karnataka, and Tamil Nadu.
The VoWiFi technology allows users to make outgoing calls to any mobile phone number and landline by connecting to a public WiFi hotspot or any private home WiFi network. The service enables no cost calling across operators and minimal data usage using a handset which supports the technology.
VoWiFi will allow telcos to take on over-the-top players like WhatsApp and keep them from eating into their revenue, even as voice calling rates rise

22. Solution: c)
The Kaleshwaram project is an off-shoot of the original Pranahitha-Chevella Lift Irrigation Scheme.
It is on the Godavari River.
The Centre under the Andhra Pradesh Reorganisation Act, 2014 is mandated to support programmes for the development of backward areas in the successor States, including expansion of physical and social infrastructure.

23. Solution: a)
Operation Twist is a move in which a central bank decides to simultaneously buy long-dated securities while selling short-term securities. The objective behind such an operation is management of the yield curve.
Other central banks, including the US Federal Reserve, have used similar measures. This is the first time RBI has undertaken such an unconventional policy measure with the aim of flattening the yield curve by lowering longer rates to boost lending and growth.
A conventional Operation Twist involves a central bank buying and selling securities of the same amounts.

24. Solution: d)
Cost of production (CoP) is one of the important factors in the determination of MSP of mandated crops. Besides cost, the Commission for ‘Agricultural Costs and Prices’ (CACP), considers other important factors such as demand and supply, price trend in the domestic and international markets, inter-crop price parity, terms of trade between agricultural and non-agricultural sectors and the likely impact of MSPs on consumers, in addition to ensuring rational utilization of natural resources like land and water.
Thus, pricing policy is rooted not in ‘cost plus’ approach, though cost is an important determinant of MSP.

25. Solution: c)
The deadline for the government’s Accessible India campaign that aims at making public spaces friendly for persons with disabilities has been extended due to slow progress. The revised deadlines have been extended to March 2020.
Department of Empowerment of Persons with Disabilities (DEPwD) has launched Accessible India Campaign (Sugamya Bharat Abhiyan) as a nation-wide Campaign for achieving universal accessibility for Persons with Disabilities (PwDs).  It has the following three important components:
Part A: Built Environment Accessibility
These would include not only buildings, but also footpaths, curb cuts, and obstacles that block the flow of pedestrian traffic.
Part B: Transportation System Accessibility
The term transportation covers a number of areas including air travel, buses, taxis, and trains.
Part C: Information and Communication Eco-System Accessibility
Access to information refers to all information. This can range from actions such as being able to read price tags, to physically enter a hall, to participate in an event, to read a pamphlet with healthcare information, to understand a train timetable, or to view webpages.

26. Solution: c)
The Ministry of Statistics and Programme Implementation (MoSPI) has tied up with Common Service Centres Network (CSCN), a special purpose vehicle under the Ministry of Electronics and IT, to carry out the census.
“For the first time, the entire census is being conducted on a digital platform by the use of an application which will ensure high accuracy and data security,”
The national economic census is conducted every five years and is crucial for framing of policies and planning for the government and other organisations.
The process of the economic census was first held in 1978. This is the seventh census which will provide disaggregated information on various operational and structural aspects of all establishments in the country.
The census will provide valuable insights into geographical spread and clusters of economic activities, ownership pattern and persons engaged of the establishments.

27. Solution: a)
 Wipro has partnered with NASSCOM (National Association of Software and Services Companies) to launch a skilling platform called ‘Future Skills’ for 10,000 students from over 20 engineering colleges in India.
This is a part of Wipro’s Corporate Social Responsibility programme, TalentNext.
TalentNext aims to enhance the quality of engineering education by preparing faculty and academic leaders to train students.
The programme has now been extended to students directly through Future Skills.
It is a new age platform built to bridge the industry-academia skill gap and help students keep pace with the emerging technologies — artificial intelligence, big data, cloud computing, cybersecurity and internet of things (IoT) – to make them future-ready.

28. Solution: c)
Chabahar Port is a seaport in Chabahar located in southeastern Iran, on the Gulf of Oman. It serves as Iran’s only oceanic port, and consists of two separate ports named Shahid Kalantari and Shahid Beheshti.
Diplomats from India, Iran and Afghanistan took stock of the activities at the Chabahar Port in South-Eastern Iran one year after its inauguration.
The Chabahar Port commenced operations almost a year ago in December 2018. Since then, we have had as many as 4,500 containers moving from India through Chabahar. Almost half a million tonne of cargo has transitted since the port was inaugurated

29. Solution: c)
Right to decent environment including pollution free water and air and protection against hazardous industries is part of Article 21.
While water supplies, irrigation and canals, drainage and embankments and storage fall in the State List, issues like development of inter-state rivers come under the Union List.
30. Solution: c)
Cybercriminals have found an under investigated vulnerability to breach Android devices. It is called StrandHogg, and it can allow them to listen to microphone, steal login credentials, take photos using camera, read SMS and even access photos. First reported by Norway-based cyber security firm Promon and later confirmed by their partner firm Lookout, the vulnerability has now caught the eye of the cyber security wing of Ministry of Home Affairs.

31. Solution (a)

32. Solution (c)
The Central Reserve Police Force (CRPF) is the premier central police force of the Union of India for internal security. Originally constituted as the Crown Representative Police in 1939, it is one of the oldest Central para military forces (now termed as Central Armed Police Force). CRPF was raised as a sequel to the political unrest and the agitations in the then princely States of India following the Madras Resolution of the All-India Congress Committee in 1936 and the ever-growing desire of the Crown Representative to help the vast majority of the native States to preserve law and order as a part of the imperial policy.
The Parliament Duty Group is an elite CRPF unit tasked with providing armed protection to Parliament House. It comprises 1,540 personnel drawn from various units of Central Reserve Police Force (CRPF). PDG members are trained in combating nuclear and bio-chemical attacks, rescue operations and behavioural management.

33. Solution (d)
Members Apart from Asian and African countries.
Europe – Albania
South America – Guyana, Suriname
A few countries with significant Muslim populations, such as Russia and Thailand, sit as Observer States.

34. Solution (a)
The Drake Passage or Mar de Hoces—Sea of Hoces—is the body of water between South America’s Cape Horn, Chile and the South Shetland Islands of Antarctica. It connects the southwestern part of the Atlantic Ocean (Scotia Sea) with the southeastern part of the Pacific Ocean and extends into the Southern Ocean.

35. Solution (b)
Russia’s military deployed a new intercontinental weapon, the Avangard hypersonic missile system that can fly 27 times the speed of sound. This will be the Russian military’s first Avangard hypersonic intercontinental ballistic missile (ICBM).

36. Solution (a)
The erstwhile Bombay Presidency, a multilingual province, included the present-day Karnataka districts of Bijapur, Belgaum, Dharwar and Uttara-Kannada (previously North Kanara).
In 1948, the Belgaum municipality requested that the district, having a predominantly Marathi-speaking population, be incorporated into the proposed Maharashtra state.
However, the States Reorganisation Act of 1956, which divided states on linguistic and administrative lines, made Belgaum a part of the then Mysore State (which was renamed Karnataka in 1973).
The Maharashtra government contested the inclusion and lodged a protest with the Centre in September 1957. This led to the formation of the Mahajan Commission under former Chief Justice Mehr Chand Mahajan in October 1966.
The Commission, which submitted its report in August 1967, recommended that 264 villages be transferred to Maharashtra and that Belgaum and 247 villages remain with Karnataka. Maharashtra rejected the report and demanded another review.
37. Solution (a)
The Andaman and Nicobar Islands were under Japanese occupation during World War II for three years (1942-45), and were formally handed over to Bose’s Azad Hind government on December 29, 1943.

38. Solution (a)
The Aṣṭādhyāyī is the central part of Pāṇini’s grammar, and by far the most complex. The Ashtadhyayi is the oldest linguistic and grammar text of Sanskrit surviving in its entirety, and Pāṇini refers to older texts and authors such as the Unadisutra, Dhatupatha, and Ganapatha some of which have only survived in part. It complements the Vedic ancillary sciences such as the Niruktas, Nighantus, and Shiksha.[60] Regarded as extremely compact without sacrificing completeness, it would become the model for later specialist technical texts OR sutras.

39. Solution (a)
The Mānasollāsa, also known as Abhilashitartha Chintamani, is an early 12th-century Sanskrit text composed by the Kalyani Chalukya king Someshvara III, who ruled in present-day South India.

40. Solution (a)

41. Solution (a)
It is a national park in Kerala. It is located in the Nilgiri hills.
The national park is one of the last undisturbed tracts of South Western Ghats mountain rain forests and tropical moist evergreen forest in India. Contiguous with the proposed Karimpuzha National Park (225 km2) to the north and Mukurthi National Park (78.46 km2) to the north-east, it is the core of the Nilgiri Biosphere Reserve (1,455.4 km2), and is part of the Nilgiri Sub-Cluster (6,000+ km2), Western Ghats World Heritage Site, recognised by UNESCO in 2007.

42 Solution (c)
While the overall green cover has increased in the country, the forest cover in the north-east — particularly in Mizoram, Arunachal Pradesh and Nagaland — has decreased.

43. Solution (c)
Protocooperation is where two species interact with each other beneficially; they have no need to interact with each other – they interact purely for the gain that they receive from doing this. It is not at all necessary for protocooperation to occur; growth and survival is possible in the absence of the interaction. The interaction that occurs can be between different kingdoms.
The flowers of plants that are pollinated by insects and birds benefit from protocooperation. The plants, particularly those with large bright colourful flowers bearing nectar glands, experience cross pollination because of the insects activities. This is beneficial to the insect that has got the food supply of pollen and nectar required for its survival.

44. Solution (d)
Talley Valley Wildlife Sanctuary is a wildlife sanctuary in Arunachal Pradesh.

45. Solution (d)
The Solar and Heliospheric Observatory (SOHO) is a spacecraft built by a European industrial consortium led by Matra Marconi Space (now Airbus Defence and Space) that was launched on a Lockheed Martin Atlas II AS launch vehicle on December 2, 1995 to study the Sun. SOHO has also discovered over 3,000 comets. It began normal operations in May 1996. It is a joint project of international cooperation between the European Space Agency (ESA) and NASA. Originally planned as a two-year mission, SOHO continues to operate after over 20 years in space: the mission is extended until the end of 2020 with a likely extension until 2022. As of 2019, only the LASCO coronagraph is still routinely producing data, with the other instruments all having been switched off.

46. Solution (a)
Source: https://www.business-standard.com/article/pti-stories/hope-peace-along-frontier-with-china-will-eventually-lead-to-resolution-of-border-dispute-army-chief-120010100794_1.html
47. Solution (c)
Gaganyaan is an Indian crewed orbital spacecraft that is intended to send 3 astronauts to space for a minimum of seven days by 2022, as part of the Indian Human Spaceflight Programme. The spacecraft, which is being developed by the Indian Space Research Organisation (Isro), consists of a service module and a crew module, collectively known as the Orbital Module. It will be for the first time that India will launch its manned mission to space, making the country fourth in line to have sent a human to space. Isro’s Geosynchronous Satellite Launch Vehicle GSLV Mk III, the three-stage heavy-lift launch vehicle, will be used to launch Gaganyaan as it has the necessary payload capability.
GSLV Mk III is designed to carry 4 ton class of satellites into Geosynchronous Transfer Orbit (GTO) or about 10 tons to Low Earth Orbit (LEO). The powerful cryogenic stage of GSLV Mk III enables it to place heavy payloads into LEO’s of 600 km altitude. The launcher uses two S200 solid rocket boosters to provide the huge amount of thrust required for lift off.

48. Solution (c)
Long March 5 (LM-5, CZ-5, or Changzheng 5) is a Chinese heavy lift launch system developed by China Academy of Launch Vehicle Technology (CALT).
49. Solution (a)
RBI launched a mobile app to help visually-impaired people to identify the denomination of currency notes.
The central bank also said the app does not authenticate a note as either genuine or counterfeit.

50. Solution (a)
51. Solution (d)
The northeast monsoon brings rain to just five of the 36 meteorological divisions in the country — Tamil Nadu (which includes Puducherry), Kerala, Coastal Andhra Pradesh, Rayalaseema and South Interior Karnataka.
52. Solution (a)
Palau is an island country located in the western Pacific Ocean. The country contains approximately 340 islands, and together with parts of the Federated States of Micronesia, forms the western chain of the Caroline Islands.
Palau has become the first country to ban sun cream that is harmful to corals. The country has banned common ingredients used in cosmetics including oxybenzone.
53. Solution (c)
For seamless and effective dissemination of emergency information and communication on disaster warnings, Potential Fishing Zones (PFZ) and Ocean States Forecasts (OSF) to fishermen, the Government launched the Gagan Enabled Mariner’s Instrument for Navigation and Information (GEMINI) device.
Indian National Centre for Ocean Information Services (INCOIS), an autonomous body under the Ministry of Earth Sciences (MoES) joined hands with Airports Authority of India (AAI) to utilize the GAGAN (GPS Aided Geo Augmented Navigation) satellite system to transmit the PFZ, OSF and disaster warnings to fishermen with GAGAN system consisting of three geosynchronous satellites (GSAT-8, GSAT-10 and GSAT-15).

54. Solution (b)
Cyclonic Storm ‘PABUK’ over Andaman Sea & neighbourhood, Cyclone ‘FANI’ over Southwest Bay & adjoining Southeast of Bengal, Cyclone Alert for Odisha, West Bengal & Srikakulam and Vijayanagaram Districts of Andhra Pradesh Coasts, Cyclonic Storm ‘VAYU’ over Northeast & adjoining East central Arabian Sea, Depression over northwest Bay of Bengal off north Odisha – West Bengal coasts, Cyclonic Storm ‘HIKAA’ over northeast and adjoining east central Arabian sea and Cyclonic Storm ‘BULBUL’ over northwest and adjoining West-Central Bay of Bengal. Other alerts like Snowfall over hills, hailstorms and Western Disturbance were also disseminated by issuing press releases.

55. Solution (a)
Nageswaraswamy Temple, Kumbakonam is a Hindu temple dedicated to Lord Shiva located in Kumbakonam, Tamil Nadu. The temple is incarnated by the hymns of Thevaram and is classified as Paadal Petra Sthalam. Shiva in the guise of Nagaraja, the serpent king and is located in the centre of Kumbakonam.
Aditya Chola constructed this temple during the 9th century. It is great marvel of Chola architecture, building technology and astronomy.

56. Answer: d
Explanation:
The Eat Right India Movement is a year-long social and mass media campaign of the Food Safety and Standards Authority of India (FSSAI). It is the new healthy eating approach which places citizens at the centre of a Health Revolution through food and fitness.
The strength of the ‘The Eat Right Movement’ lies in its holistic and collaborative approach, with stakeholders on both the demand and supply-side joining to make a difference through some clearly identified steps.
On the demand side, the Eat Right Movement focuses on empowering citizens to make the right food choices.
On the supply side, it nudges food businesses to reformulate their products, provide better nutritional information to consumers and make investments in healthy food as responsible food businesses.
‘The Eat Right Movement’ brings together three ongoing initiatives of FSSAI:
Safe and Nutritious Food Initiative, focused on social and behavioural change around food safety and nutrition at home, school, workplace and on-the-go;
The Eat Healthy Campaign focused on reduction of high fat, sugar and salt foods in the diet; and
Food Fortification, focused on promoting five staple foods – wheat flour, rice, oil, milk and salt that are added with key vitamins and minerals to improve their nutritional content.
57. Answer: b
Explanation:
‘Miyawaki forests’ is a highly successful technique, pioneered by Japanese botanist Akira Miyawaki.
The concept involves raising indigenous and native trees in dense plantations mimicking forest settings. It involves the preparation of the substrate if the land involved is much degraded.
It promotes natural vegetation on degraded land.
58. Answer: d
Explanation:
The insurance cover provided under the PM-JAY health insurance scheme is 5 lakh per family.
The PM Jan Arogya Yojana beneficiaries get an e-card that can be used to avail services at an empanelled hospital, public or private, anywhere in the country.
The scheme will come to the aid of economically disadvantaged families as per data in the Socio-Economic Caste Census 2011.  PMJAY will benefit urban workers’ families in the occupational category present as per the Socio-Economic Caste Census 2011.
59. Answer: a
Explanation:
Arranging the cities from north to south: Brisbane, Perth, Sydney, Canberra, Melbourne.
60. Ans: c
Explanation:
Special provisions have been provided to some states under the Constitution of India, listed in Articles 371 and 371(A-J). While Articles 370 and 371 have been a part of the Constitution since January 26, 1950, Articles 371(A-J) were incorporated through amendments under Article 368, which lays down the power of the Parliament to amend the Constitution and procedure therefor.
Maharashtra, Gujarath, Nagaland, Assam, Manipur, Present Telangana, Sikkim, Mizoram, Arunachal Pradesh, Goa and Karnataka have provisions of special status under Article 371 of the Constitution.

61. Answer: c

62. Answer: b
Explanation:
India presently has councils for each of the five zones: North, Central, Eastern, Southern and Western.
The five zonal councils created under the States Re-Organization Act, 1956, are advisory bodies.
The North-Eastern states are not covered by any of the Zonal Councils and their special problems are addressed by another statutory body, the North Eastern Council, created by the North Eastern Council Act, 1971.

63. Answer: a
Explanation:
Arranging the ports from north to south:
Vishakapatnam port
Mormugao port
Krishnapatnam port
Ennore port
Mangalore port

64. Answer: d
Explanation: The Bureau of Energy Efficiency is an agency of the Government of India, under the Ministry of Power. It was created in March 2002 under the provisions of the nation’s 2001 Energy Conservation Act.

65. Answer: a
Explanation: After the second Anglo-Maratha war, Shinde and Holkar had lost many of their territories to the British. They encouraged the Pindaris to raid the British territories. The Pindaris, who were mostly cavalry, came to be known as the Shindeshahi and the Holkarshahi after the patronage they received from the respective defeated Maratha leaders. The total strength of the Pindaris in 1814 was estimated at 33,000. The Pindaris frequently raided villages in Central India. The result of the Pindari raids was that Central India was being rapidly reduced to the condition of a desert because the peasants were unable to support themselves on the land. They had no option but to join the robber bands or starve. In 1815, 25,000 Pindaris entered the Madras Presidency and destroyed over 300 villages on the Coromandel Coast. Another band swept the Nizam’s kingdom while a third entered Malabar. Other Pindari raids on British territory followed in 1816 and 1817. Francis Rawdon-Hastings saw that there could not be peace or security in India until the predatory Pindaris were extinguished and launched an operation against them during the Third Anglo-Maratha War.

66. Answer: c
Explanation: Article 243K which provides for elections to the panchayats states that – the superintendence, direction and control of the preparation of electoral rolls for, and the conduct of, all elections to the Panchayats shall be vested in a State Election Commission consisting of a State Election Commissioner to be appointed by the Governor.

67. Answer: c
Explanation: Self-explanatory

68. Answer: c

69. Answer : b
National Initiative for School Heads’ and Teachers’ Holistic Advancement (NISHTHA) 
It aims to improve learning outcomes at the elementary level through an Integrated Teacher Training Programme.
This integrated programme aims to build the capacities of around 42 lakh participants covering all teachers and Heads of the school at the elementary level in all government school, faculty members of states councils of Educational Research and Training and other educational departments.
The integrated training will motivate and equip teachers to encourage and foster critical thinking in students, handle diverse situations and act as first-level counsellors.
Pradhan Mantri Innovative Learning Program (DHRUV) aims to identify and encourage talented children to enrich their skills and knowledge.
It act as a platform  to explore the talent of outshining and meritorious students, and help them achieve excellence in their specific areas of interest may it be science, performing arts, creative writing, etc.
These talented students apart from realizing their full potential are expected to contribute to the society in a big way.
With children drawn from all over the country, the DHRUV programme reflects the true spirit of EK Bharat Shreshth Bharat.

70. Answer : b
Patola, the trademark Saree of Gujarat, is considered to be very costly and worn only by the Royals or the Aristocrat.
Reason being the raw material silk yarn is purchased from Karnataka or West Bengal, where silk processing units are situated, thus increasing the cost of the fabric manifolds.
In a historic initiative taken by Khadi and Village Industries Commission (KVIC), a first Silk Processing Plant was inaugurated recently at Surendranagar in Gujarat.
It would help cut down the cost of production of silk yarn drastically and increase the sale and availability of raw material for Gujarati Patola Sarees locally.

71. Answer : c
The restoration work of Markandeshwar temple in Maharashtra   by the Archaeological Survey of India is in full swing.
Known as the “Khajuraho of Vidarbha”, the temple of Markandadeo is situated on the bank of River Wainganga in district Gadchiroli of Maharashtra.
The temples belong to the Nagara group of temples of North India.
72. Answer : a
E-visa is an online process where there is no paper work is involved and all the process happens online.
e-Visa is valid for entry through 28 designated Airports and 5 designated seaports.
At present the e-Visa Scheme is available to nationals of 169 countries.
One can’t apply for e-visa at the airport. One must hold a valid e-visa before leaving one’s home country.
eVisa is admissable only under the following categories:
e-Tourist Visa
e-Business Visa
e-Conference Visa
e-Medical Visa
e-Medical Attendent Visa

73. Answer : b
To objectively evaluate the performance of the school education system in the States/UTs, Ministry of Human Resources and Development (MHRD) has designed a 70 indicators based matrix called Performance Grading Index (PGI) to grade the States and UTs.

74. Answer : b
WHOalong with the International Confederation of Midwives (ICM), International Council of Nurses (ICN), Nursing Now and the United Nations Population Fund (UNFPA), designated 2020 as the International Year of the Nurse and the Midwife.
The world will need an additional 9 million nurses and midwives to achieve the commitment of providing all people with access to health care by 2030, WHO has warned.
World Drug Reportis published byUnited Nations Office on Drugs and Crime (UNODC).
World Health Statistics, World Tuberculosis Report, Ambient Air Pollution Reportare few important reports published by WHO.

75. Answer : a
EVALI (e-cigarette, or vaping, product use associated lung injury) is a new name given to Vaping-Related Illness.
The U.S. Centers for Disease Control and Prevention noted the name in new guidance on the illness.
E-cigarettes, also called ‘vapes’ or ‘electronic nicotine delivery systems (ENDS)’, are battery-run devices that were originally marketed as a safer alternative to smoking traditional cigarettes.
In recent years, there has been a deadly rise in nicotine addiction,allegedly due to aggressive marketing by manufacturers.
Minors have been particularly affected in US, with a 2018 survey showing that as many one in five and one in 20 students going to high school and middle school respectively using e-cigarettes.
In September 2019, the Indian government banned the production, import, distributionand sale of electronic cigarettes.
76. Answer : a
Recently, Securities and Exchange Board of India(SEBI) appoints G Babita Rayudu as the Executive Director.
The Securities and Exchange Board of India (SEBI) was established on April 12, 1992 in accordance with the provisions of the Securities and Exchange Board of India Act, 1992.
It is a statutory body.
SEBI is administered by its board of members. The board of SEBI consist of,
The Chairman by nominated by Government of India.
2 members from Finance ministry
1 member from Reserve Bank of India
5 members nominated by Union Government of India
While trying any suit, SEBI has the same power as vested with a Civil court under the Code of Civil Procedure, 1908.
77. Answer : c
For the first time, researchers have shown that a species of Brittle stars, which are relatives of starfish, can see even though it does not have eyes.
The ability to see without eyes is known as Extraocular Vision.
The Red Brittle star (Ophiocomawendtii), which lives in the coral reefs of the Caribbean Sea, becomes only the 2ndcreature, after a Sea urchin species, known to have this ability (barring freak cases in other species).
Researchers suspect that Extraocular vision is facilitated by the Photoreceptor cells found on their bodies.
The Brittle star sees with the help of light-sensing cells that cover its entire body.
These light-sensing cells give the brittle star visual stimuli, allowing it to recognise coarse structures such as rocks, the research suggests.
78. Answer : c
A first-of-its-kind Rehabilitation centre for Freshwater Turtles will be inaugurated in Bihar’s Bhagalpur forest division in January 2020.
The rehabilitationcentre,will be able to shelter 500 turtles at a time.
The centre will house the injured and the sick turtles rescued from smugglers.
Eastern Bihar has been an ideal breeding ground for turtles, in Bhagalpur, the flow of water in the Ganga is ample.
Also, there are many sandbanks in the middle of the river, which are ideal breeding ground for turtles.
According to environmentalists, the turtles play a significant role in the river by,
Scavenging dead organic materials and diseased fish,
Controlling fish population as predators and
Controlling aquatic plants and weeds.
They are also described as indicators of healthy aquatic ecosystems.
According to a recent study conducted byTraffic India, around 11,000 turtles are being smuggled in India every year.
In the past 10 years, as many as 110,000 turtles have been traded
79. Answer : a
Nepal has raised objections after India released its new political map in November 2019 following the reorganisation of Jammu and Kashmir.
Nepal claimed that Limpiyadhura, Lipulek and Kalapani areas were shown under India’s territory even though they lie within the Nepalese territory.
80. Answer : c
The Anglo-Nepalese War is also known as the Gurkha War.
It was fought between the Kingdom of Gorkha (present-day Nepal) and the East India Company as a result of border disputes and ambitious expansionism of both the belligerent parties.
The war ended with the signing of the Treaty of Sugauli in 1816, which ceded some Nepalese controlled territory to the British.
Under the treaty, the Nepalese-controlled territory that was ceded included all areas that the king of Nepal had won in earlier wars such as the kingdom of Sikkim in the east and Kumaon and Garhwal in the west.
81. Answer : d
Savitribai Phule was the wife of Jyotirao Phule, an Indian activist, thinker, social reformer and writer from Maharashtra.
She was determined to study and was one of the very few indigenous literate women in her era.
Savitribai, along with her husband Jyotirao Phule, stood up for the rights of women and fought against the injustice faced by them.
They were the pioneers of women education in India and started the first girls’ school in 1848 in Pune.
Their work extended to many fields including eradication of untouchability and the caste system, women's emancipation and the reform of Hindu family life.
Savitribai Phule started Mahila Seva Mandal in 1852, which worked for raising women’s consciousness about their human rights, dignity of life and other social issues.
First ever infanticide prohibition home of India was started by Savitribai Phule in 1853.
They championed widow remarriage and started a home for lower and upper caste widows in 1854.
In September 1873, Phule, along with the followers, formed the Satyashodhak Samaj (Society of Seekers of Truth) to attain equal rights for peasants and people from lower castes.
82.Answer : c
Maharashtra Government has launched a ‘Cyber Safe Women’ initiative under which awareness camps will be held across all the districts of the state regarding cyber safety.
The initiative will help in educating women about how the web is used by anti-social elements and child predators to commit various types of crimes.
83.Answer : a
In Tripura, Lai Haraoba, a ritualistic festival observed by Manipuri meitei communities since ancient times, began recently.
The five day long festival is jointly organised by Department of Information and Cultural Affairs, Government of Tripura, Puthiba Lai Haraoba Committee and Puthiba Welfare & Cultural Society, Agartala.
The festival aims to uphold tradition and cultural values of Meitei community.
84.Answer : a
Madras Stock Exchange (MSE) strikes the gong in new avatar  with the Mobile app DailyGong offers a gamut of financial products.
Plans are also afoot to bring mutual funds, fixed deposits under DailyGong in the near future.
While digital transformation rendered it irrelevant earlier, MSE in its new avatar has leveraged the same technology to serve investors. If it gets its act right, it may even regain its lost glory.
A bull market is a  financial market of a group of securities in which prices are rising or are expected to rise.
Bull markets are characterized by optimism, investor confidence and expectations that strong results will continue.
Bear market refers to a market condition in which the prices of securities are falling, and widespread pessimism causes the negative sentiment to be self-sustaining.
Along with BSE (Bombay Stock Exchange) and NSE (National Stock Exchange) there are other Stock Exchanges functioning in India.
According to SEBI the above listed are the Stock Exchanges functioning in India and few others are closed recently.
85.Answer : b
ISRO has decided to send 4 air force pilots to Russia to train them as astronauts for the country’s maiden human spaceflight, Gaganyaan.
The spacecraft is being developed by the Indian Space Research Organisation (ISRO).
The Gaganyaan programme, an indigenous mission that would take Indian astronauts to space.
It consists of a Service module and a Crew module, collectively known as the Orbital Module.
The Crew module house the astronauts and a Service module maintains the speed and orientation of the spacecraft.
ISRO's GSLV Mk III, the three-stage heavy-lift launch vehicle, will be used to launch Gaganyaan.
The programme will make India the 4th nation in the world to launch a Human Spaceflight Mission.
So far, only the USA, Russia and China have launched human spaceflight missions.
86.Answer : c
Recently, the Northeast, or Winter monsoon has ended on a high, with an overall surplus rainfall being recorded for the season.
ITCZ (Intertropical Convergence Zone) is a zone between the northern and southern hemisphere where winds blowing equator-ward from the mid latitudes and winds flowing poleward from the tropics meet.
It shifts from north and south seasonally according to the movement of the Sun.
When the ITCZ is shifted to north of the Equator, the southeast trade wind changes to a southwest wind as it crosses the Equator.
The Indian climate is characterized by the complete reversal of wind system with the change of season in a year.
During the winter season winds generally blow from north-east to south-west in the direction of trade winds.
These winds are dry, devoid of moisture and are characterized by low temperature and high pressure conditions over the country.
During summer season complete reversal in the direction of the winds is observed and these blow primarily from south-west to north-east.
87.Answer : c
RBI launched the “Mobile Aided Note Identifier (MANI)”, a mobile application for aiding visually impaired persons to identify the denomination of Indian Banknotes.
MANI, has the following features:
Capable of identifying the denominations of Mahatma Gandhi Series and Mahatma Gandhi (New) series banknote by checking front or reverse side/part of the note including half folded notes at various holding angles and broad range of light conditions (normal light/day light/low light/ etc.).
Ability to identify the denomination through audio notification in Hindi/English and non-sonic mode such as vibration (suitable for those with vision and hearing impairment).
After installation, the mobile application does not require internet and works in offline mode.
Ability to navigate the mobile application via voice controls for accessing the application features wherever the underlying device & operating system combination supports voice enabled controls.
The application is free and can be downloaded from the Android Play Store and iOS App Store without any charges/payment.
This mobile application does not authenticate a note as being either genuine or counterfeit.
88.Answer : d
US kills top Iranian general in Baghdad airstrike.
Qassem Soleimani, head of Iran's elite Quds Force, killed in an air strike as tensions between US and Iran escalate.
Iran has one of the longest land borders of any country in western Asia covering 3,662 miles in length.
There are 7 countries that Iran shares this long land border, these countries are Iraq, Turkmenistan, Afghanistan, Pakistan, Turkey, Armenia, and Azerbaijan.
Of these countries, Turkmenistan shares the longest border with Iran.
Kuwait and Syria do not share borders with Iran.

89.C.Both 1 and 2
Explanation :
The Dedicated Freight Corridor Corporation of India, Ltd. (DFCCIL) has began testing a new wagon that will help boost the cargo carrying capacity by four times due to increased length of the train, use of double-stack containers and more payload carrying capability. The new wagon — called BLCS (type A & B) — is currently a prototype. A total of three such wagons have been manufactured. The wagon — used for hauling freight — have been tested by the Research Design and Standards Organisation (RDSO) and are manufactured at the Golden Rock Railway Workshop in Tiruchirappalli, Tamil Nadu. These wagons will be under trial till January 26, 2020. If approved by the RDSO, wagons will be manufactured commercially as per industry requirements. Hence both statements are correct.

90.B. 2 only Explanation :

Statement 1 is incorrect:
The Forest Survey of India (FSI) released the 2019 report on Forest Fires. It is based on a study carried out by the FSI along forest fire points identified across the country from 2004 to 2017. About 21.4% of forest cover in India is prone to fires, with forests in the north-eastern region and central India being the most vulnerable. Extremely fire prone areas account for 3.89% of total forest cover, very highly fire prone areas account for 6.01% and highly fire prone areas for 11.50%. Together, the three categories come to 21.40 % of forest cover. Central Indian States also recorded a high number of forest fire alerts, with Madhya Pradesh accounting for 2,723 alerts; Maharashtra 2,516; Odisha 2,213 and Chattisgarh 1,008 alerts between November 2018 to June 2019.
91.A. 1 only
Explanation :
Statement 1 is correct:
According to an order recently issued, the Railways renamed the Railway Protection Force (RPF) as Indian Railway Protection Force Service and accorded it organised Group A status (OGAS). Grant of status of Organized Group 'A' service to RPF will end stagnation, improve career progression of the officers and keep up their motivational level. Background: In 2012, High Court of Delhi directed Railways to grant Group 'A' Service status to RPF. Same was upheld by Supreme Court of India in 2019. Accordingly, Railway Board had proposed for grant of Organised Group 'A' Service status to RPF.

92.C.Both 1 and 2
Explanation :
The Chief of the Air Staff formally inducted the Flight Inspection system (FIS) Dornier 228 aircraft into No 41 Squadron, the ‘Otters.’ The Dornier-228 aircraft is a multi-purpose, fuel efficient, rugged, light weight twin turboprop aircraft with a retractable tricycle landing gear. The aircraft's cockpit is designed to accommodate two crew members and is fitted with duplicate controls. The modified Dornier 228 aircraft have been acquired to undertake in-house calibration of Navigational aids available after implementation of Modernised Airfield Infrastructure (MAFI) at IAF bases. The Indian Air Force (IAF) in 2015 had signed a contract to purchase 14 Dornier aircraft, which are flight inspection system planes, from state-run Hindustan Aeronautics Limited (HAL). While the first aircraft was delivered in November 2019, the second is expected to be delivered in early 2020. This aircraft has been exported to Seychelles and Mauritius. Hence both statements are correct.

93.A. Andhra Pradesh
Explanation :
The Belum Caves Festival in Andhra Pradesh, which was to be organised in December 2019 will now be held this month (January 2020) to highlight the importance of the Balum Caves. The Belum Caves are located near Belum Village in Kurnool District, Andhra Pradesh. It is the second largest caves on the Indian Subcontinent after the Krem Liat Prah caves in Meghalaya. This cave system is known for its speleothems, such as stalactite and stalagmite formations. It has long passages, galleries, spacious caverns with fresh water and siphons. The caves consist of black limestone. This cave system was formed over the course of tens of thousands of years by the constant flow of underground water from the now-disappeared river Chitravathi. Hence, option (a) is the correct answer.

94.A.   Indian classical sarod player
Explanation :
Amjad Ali Khan has relinquished his plans to set up an international residential institute in Kerala to impart training in music of different genres (Hindustani, Carnatic and Western) after Kerala’s decision to include bureaucrats in governing body of this proposed international music academy. Amjad Ali Khan (born 1945) is an Indian classical sarod player. He is a sixth-generation descendant of the Bangash gharana of sarod players; his family claims to have invented the sarod. He is son of Hafiz Ali Khan (1888–1972), an Indian sarod player and recipient of Padma Bhushan. He is best known for his clear and fast ekhara taans. Awards and honours received: Padma Vibhushan, Banga-Vibhushan, Sangeet Natak Akademi Award, Sangeet Natak Akademi Fellowship and and Rajiv Gandhi National Sadbhavna Award. Hence, option (a) is the correct answer.

95.C. Both 1 and 2
Explanation :
Indian Railways unveiled Him Darshan Express comprising all-new Vistadome coaches from Kalka railway station to Shimla. Commuters travelling in Him Darshan Express will enjoy the beautiful scenic view of the Himalayas as the train will have Vistadome AC coaches with glass roof. This special train will commute between the Kalka and Shimla station for the next one year till December 24, 2020. This is a first-of-its-kind train on the Kalka-Shimla route. Him Darshan Express will be the first-ever train with Vistadome coaches by the Indian Railways that will run on a regular basis. Vistadome Coach consists of a glass roof which has electrically controlled opalescence, that is, it can be made transparent or opaque at the turn of a switch. The first Vistadome Coach was commissioned by the Ministry of Railways on the Vishakhapatnam – Araku Valley Route in April, 2017. Hence both statements are correct.

96.B. 2 only
Explanation :
As of December 27,2019 the Centers for Disease Control and Prevention (CDC), a US federal agency, has reported 2,561 cases across the country who have suffered EVALI, the mysterious respiratory illness linked to vaping and e-cigarettes. EVALI is the name given by the Centers for Disease Control and Prevention (CDC) to the dangerous, newly identified lung disease linked to vaping. The name EVALI is an acronym that stands for e-cigarette or vaping product use-associated lung injury. The illness was first recognized by the CDC in August 2019 after health department officials across the country began to work together to study cases of severe, sometimes fatal, lung infections that arose suddenly in otherwise healthy individuals. It’s therefore unclear how the condition develops or why, in the most severe and life-threatening cases, it causes the lungs to stop functioning altogether. CDC urge everyone to avoid e-cigarette or vaping products that contain tetrahydrocannabinol, or THC (a high-inducing chemical derived from marijuana). Hence only statement 2 is correct.

97.A. World Economic Forum
Explanation :
Government is committed to make policies and programmes for the development and promotion of tourism and the year 2019 witnessed several steps in this direction. In the year 2019, India’s rank in Travel and Tourism Competitiveness Index of World Economic Forum has moved to 34th position from 65th rank in 2013. Foreign Tourist Arrivals also registered a growth of 3.2 percent last year. The government is working to develop thematic circuits under the Swadesh Darshan Scheme and 77 projects amounting over Rs 6035 crore have been sanctioned till date under this scheme. An initiative called - 'Adopt a Heritage: Apni Dharohar, Apni Pehchaan' has been launched for developing tourist amenities at heritage and tourist sites and making them tourist-friendly. E-visa has been further liberalized and visa fee on e - visa has been substantially reduced to increase tourism competitiveness of the country. The Government has also opened more than 120 mountain peaks for mountaineering and trekking for promoting adventure tourism in the country. Hence, option (a) is the correct answer.

98.C. Both 1 and 2
Explanation :
The world Braille Day was celebrated on January 4, 2020. The day is marked to remember the birth anniversary of Louis Braille, the inventor of Braille - for people with visual disabilities. World Braille Day, celebrated since 2019, is observed to raise awareness of the importance of Braille as a means of communication in the full realization of the human rights for blind and partially sighted people. Braille is a system that enables blind and visually impaired people to read and write through touch. Braille is named after its creator, Louis Braille, a Frenchman who lost his sight as a result of a childhood accident. It was devised by Louis Braille in 1821. The Braille Script consists of raised dots arranged in "cells." A cell is made up of six dots that fit under the fingertips, arranged in two columns of three dots each. Each cell represents a letter, a word, a combination of letters, a numeral or a punctuation mark. Braille is essential in the context of education, freedom of expression and opinion, as well as social inclusion, as reflected in article 2 of the Convention on the Rights of Persons with Disabilities. Hence both statements are correct.


99. Correct option: (a)
Explanation:
Statement 3 is incorrect: India was invited for
the fi rst time ever in the recent summit. It is
not a founding member of the organization.
Supplementary notes
Islamic Cooperation countries (OIC)
 The Organisation of Islamic Cooperation (OIC)
is the second largest inter-governmental
organization after the United Nations with
a membership of 57 states spread over four
continents.
The Organization was established upon a
decision of the historical summit which took
place in Rabat, Kingdom of Morocco on 12th
Rajab 1389 Hijra (September 25, 1969).
 In 1970, the fi rst-ever meeting of the Islamic
Conference of Foreign Minister (ICFM) was
held in Jeddah, which decided to establish a
permanent secretariat.
The secretariat comprises a secretary-general
who is the Chief Administrative Offi cer of the
organisation.
 The majority of its member states are Muslimmajority countries, while others have signifi cant
Muslim populations, including several African
and South American countries.
Over the last 40 years, the membership has
grown from its founding members of 30 to
57 states.
The OIC has permanent delegations to the
United Nations and the European Union. The
offi cial languages of the OIC are Arabic, English,
and French.
While the 22 members of the Arab League
are also part of the OIC, the organisation has
several signifi cant non-Arab member states,
including Turkey, Iran and Pakistan.
 It also has fi ve observer members, including
Russia and Thailand.

100. Correct option: (d)
Explanation:
Nagpur resolution is to empower the citizens
by policy interventions for better service
delivery through timely updation of citizen’s
charters, implementation of enactments and
benchmarking standards for continuous
improvement.
Supplementary notes
‘Nagpur Resolution: A Holistic approach
for empowering citizens’
Performance audit of public servants should
be done at regular intervals.
The Conference also resolved to adopt a holistic
approach of systemic public grievance reforms
through improved mapping, formulation
of monitoring matrix, data collection and
evaluation in quality of grievance redressal, and
to provide an enabling environment for States
and Ministries/Departments of the Government
of India for creating web portals and to adopt a
holistic approach for improved service delivery
through digital platforms.
The Resolution focuses on dynamic policy
making and strategic decisions, monitoring of
implementation, appointment of key personnel,
coordination and evaluation, and achieving
a sense of common identity by exchange of
technical expertise in the areas of Improved
Service Delivery between paired States under
the Ek Bharat Shreshtha Bharat Program.
To empower the citizens by policy interventions
for better service delivery through timely
updation of citizens charters, implementation
of enactments and benchmarking standards
for continuous improvement;
To empower citizens by adopting a bottom-up
approach to bring massive improvements in
quality of grievance redressal and reduction
in timelines of grievance redressal;

101. Correct option: (c)
Explanation:
Statement 2 is incorrect: It is an apex
organisation under the Ministry of Micro, Small
and Medium Enterprises, with regard to khadi
and village industries within India.
Supplementary notes
Khadi and Village Industries Commission
(KVIC)
 The Khadi and Village Industries Commission
(KVIC) is a statutory body formed by the
Government of India, under the Act of
Parliament, ‘Khadi and Village Industries
Commission Act of 1956’.

102. Correct option: (b)
Explanation:
Statement 1 is incorrect: The rankings were
launched by the Department of Administrative
Reforms and Public Grievances, and the Centre
for Good Governance.
Supplementary notes
Good Governance Index
The rankings were launched by the Department
of Administrative Reforms and Public Grievances,
and the Centre for Good Governance.
Tamil Nadu topped the Good Governance
Index. Maharashtra, Karnataka, Chhattisgarh
and Andhra Pradesh followed among the ‘Big
States’.

103. Correct option: (b)
Explanation:
Statement 1 is incorrect: Prime Minister
honoured the contribution of former Prime
Minister Atal Bihari Vajpayee by naming the
Strategic Tunnel under Rohtang Pass after
him.
Statement 3 is incorrect: It will reduce
the distance between Manali and Leh by
46 kilometres and save crores of rupees in
transport costs.
Supplementary notes
Rohtang Tunnel
The historic decision to construct a strategic
tunnel below the Rohtang Pass was taken on
June 03, 2000, when late Atal Bihari Vajpayee
was the Prime Minister.
The foundation stone for the Access Road to
the South Portal of the tunnel was laid on
May 26, 2002.
How long is the tunnel, and what is special
about it?
Upon completion, the 8.8 km-long tunnel will
be the world’s longest highway tunnel at an
altitude of above 10,000 feet (3,000 metres).
It is a 10.5 m-wide single tube, a bi-lane tunnel
with a fi reproof emergency tunnel built into the
main tunnel itself. The 10.5-m width includes
a 1-metre footpath on both sides.

104. Correct option: (b)
Explanation
 Statement 1 is incorrect: Atal Bhujal Yojana
has been launched amid growing concerns over
the country’s groundwater resources.
Statement 3 is incorrect: Implementation
of the scheme is expected to benefi t nearly
8350 Gram Panchayats in 78 districts in these
States only.
Supplementary notes
Atal Bhujal Yojana - A scheme for
groundwater
 Prime Minister launched the Atal Bhujal Yojana,
or Atal Jal.
 Atal Jal is a World Bank-funded, central
scheme aimed at improving groundwater
management.
The idea first came up in 2015, in view
of depleting groundwater resources. The
government announced its intention to start a
programme for management of groundwater
resources in the Budget of 2016-17.
ATAL JAL has been designed with the principal
objective of strengthening the institutional
framework for participatory groundwater
management and bringing about behavioral
changes at the community level for sustainable
groundwater resource management in seven
States, viz. Gujarat, Haryana, Karnataka, Madhya
Pradesh, Maharashtra, Rajasthan and Uttar
Pradesh.

105. Correct option: (d)
Explanation:
All the above statements are correct
Supplementary notes
NPR - National Population Register
The Census Commission has said the objective
of the NPR is to create a comprehensive
identity database of every “usual resident” of
the country.
The database will have demographic details.
A “usual resident”, for the NPR, is a person
who has lived in an area for at least six months
or more, or a person who intends to live in
an area for the next six months or more. It is
mandatory for every “usual resident” of India
to register in the NPR.
The NPR, since it is linked to the Census, is
seen as the fi rst step towards a nationwide
exercise to implement the National Register
of Citizens (NRC).
Though an NPR doesn’t necessarily mean it’s
guaranteed there will be an NRC, it clears the
path for a nationwide citizens’ list.
This is seen as one of the reasons why some
states like West Bengal and Kerala, which are
opposed to the NRC, have stopped work on
the NPR.
 NPR was fi rst done in 2010 and was later
updated in 2015 when it was linked with the
Aadhar.
The NPR is a register of the usual residents
of the country.
It contains information collected at the local
(village/sub-town), subdistrict, district, state
and national level under provisions of the
Citizenship Act, 1955 and the Citizenship
(Registration of Citizens and Issue of National
Identity Cards) Rules, 2003.
The process of updating NPR will be carried
out under the aegis of the Registrar General
and ex-Offi cio Census Commissioner, India.

106. Correct option: (c)
Explanation:
Both the statements are correct
Supplementary notes
JCPOA
Joint Comprehensive Plan of Action (JCPOA):
JCPOA is also known as the “Iran deal” or “Iran
nuclear deal”.
! The deal was made in July 2015.
! It is an agreement on Iran’s nuclear program
made between Iran, the P5+1 (the fi ve
permanent members of the United Nations
Security Council—China, France, Russia,
United Kingdom, United States—plus
Germany) and the European Union.
! JCPOA limits Iran’s uranium enrichment
programme until 2030 and contains
monitoring and transparency measures that
will remain in place long after that date.
! A few days after the JCPOA was agreed,
it was endorsed by the United Nations
Security Council (UNSC).
However, in May 2018, United States
announced its withdrawal from JCPOA.
Why did US withdraw from JCPOA?
United States cited major fl aw in JCPOA’s
temporary nature and its lack of controls
on Iran’s ballistic missile programme.

107. Correct option: (b)
Explanation:
 Statement 1 is incorrect: Ebola virus was fi rst
discovered in 1976 near the Ebola River in what
is now the Democratic Republic of Congo.
Supplementary notes
Ebola epidemic
Health officials in Eastern Congo have
documented the fi rst relapse of the current
Ebola epidemic.
The Ebola outbreak in Democratic Republic
of Congo has so far infected more than
3,300 people and killed more than 2,200 since
the middle of last year, making it the secondworst on record.
 According to WHO, this recent relapse
happened in the Aloya district where a
motorcycle taxi operator is said to have come
into contact with 33 people potentially the
cause of 11 new confi rmed Ebola cases in
the past week.
About Ebola
Ebola Virus Disease (EVD) is a rare and deadly
disease in people and nonhuman primates.
The viruses that cause EVD are located mainly
in sub-Saharan Africa.
People can get EVD through direct contact
with an infected animal (bat or nonhuman
primate) or a sick or dead person infected
with Ebola virus.
The US Food and Drug Administration (FDA)
has not yet approved a vaccine or treatment
for Ebola virus infection. It is caused by an
infection with a group of viruses within the
genus Ebolavirus:
! Ebola virus (species Zaire ebolavirus)
! Sudan virus (species Sudan ebolavirus)
! Taï Forest virus (species Taï Forest ebolavirus,
formerly Côte d’Ivoire ebolavirus)
! Bundibugyo virus (species Bundibugyo
ebolavirus)
! Reston virus (species Reston ebolavirus)
! Bombali virus (species Bombali ebolavirus)
 Of these, only four (Ebola, Sudan, Taï Forest,
and Bundibugyo viruses) are known to cause
disease in people.
Ebola virus was first discovered in 1976
near the Ebola River in what is now the
Democratic Republic of Congo.
It kills up to 90% of people who are
infected.

108. Correct option: (a)
Explanation:
 2nd statement is incorrect: Sambhar has been
designated as a Ramsar site.
Supplementary notes
Recently, the Indian Veterinary Research
Institute (IVRI) has attributed the deaths of
migratory birds to avian botulism at Sambhar
Lake in Rajasthan. In this context, Sambhar
lake was in news.
 The avian botulism is a neuro-muscular
illness caused by Botulinum (natural toxin) that
is produced by a bacteria — Clostridium
botulinum.
 The Sambhar Salt Lake is India's largest inland
saltwater body located near Jaipur in Rajasthan.
 The lake is surrounded on all sides by the Aravali
hills.
It is the source of most of Rajasthan's salt
production.

109. Correct option: (a)
Explanation:
 4th statement is incorrect. Bharart Darshan
Scheme does not aim at building rural
infrastructure.
Supplementary notes
Salient Features of Bharat Darshan
Scheme
100% centrally funded for the project
components undertaken for public funding.
To leverage the voluntary funding available
for Corporate Social Responsibility
(CSR) initiatives of Central Public Sector
Undertakings and corporate sector.
Funding of individual project will vary from
state to state and will be fi nalised on the
basis of detailed project reports prepared by
PMC (Programme Management Consultant).
PMC will be a national level consultant to be
appointed by the Mission Directorate.
A National Steering Committee (NSC) will be
constituted with Minister in charge of M/O
Tourism as Chairman, to steer the mission
objectives and vision of the scheme.
A Mission Directorate headed by the Member
Secretary, NSC as a nodal offi cer will help
in identifi cation of projects in consultation
with the States/ UTs governments and other
stakeholders.
Scheme Objectives

110. Correct option: (a)
Explanation:
Option (a) is correct.
Supplementary notes
Recently, the Ministry of Environment, Forest
and Climate Change (MoEFCC) organized
an annual meeting of state nodal agencies
implementing the ‘Eco club’ programme for
the fi rst time.
 National Green Corps is a programme started
in 2001-02 by the Ministry of Environment
Forests and Climate Change.
Aim: To provide opportunities for children to
understand the environment and environmental
problems through school eco-clubs.
 The programme is a sub part of Environment
Education Awareness and Training (EEAT) which
is a central sector scheme of the Ministry of
the Environment continuing since 1983-84.
Objectives of the Programme
To impart knowledge to school children
through hands-on experience, about their
immediate environment, interactions within it
and the problems therein.
To develop requisite skills of observation,
experimentation, survey, recording, analysis
and reasoning for conserving the environment
through various activities.
To inculcate the proper attitude towards the
environment and its conservation through
community interactions.
To sensitize children to issues related to
environment and development through fi eld
visits and demonstrations.
To promote logical and independent thinking
among children so that they are able to
make the right choices in a spirit of scientifi c
inquiry.

111. Correct option: (c)
Explanation:
Both the statements are correct.
Supplementary notes
Project Tiger and National Tiger
Conservation Authority (NTCA)
Project Tiger was launched in 1973 with 9 tiger
reserves for conserving our national animal,
the tiger. Currently, the Project Tiger coverage
has increased to 50, spread out in 18 tiger
range states.

112. Correct option: (d)
Explanation:
Statements 1 and 2nd are correct. Only
third statement is incorrect. Quick Reaction
Surface to Air Missile (QRSAM) system will be
commissioned to Indian Army.
Supplementary notes
DRDO successfully test-fi red its Quick Reaction
Surface to Air Missile (QRSAM) system, likely
to be inducted into the armed forces by 2021,
from a base off Odisha coast.
 The missile, developed by the Defence Research
and Development Organisation (DRDO) for
the Indian Army, was fl ight-tested from the
Integrated Test Range (ITR) at Chandipur
It has been developed to replace the ‘Akash’
missile defence system, and has 360-degree
coverage.
 The fi rst test fi ring of the missile took place
on 4 June 2017. This was followed by the
second successful test on 3 July 2017.
 The test fl ights had successfully demonstrated
their aerodynamics, propulsion, structural
performance and high maneuvering
capabilities.
Features
! It uses solid fuel propellant and has a strike
range of 25-30 km with capability of hitting
multiple targets.
! Developed by DRDO
! It is capable of hitting the low flying
objects.
In the past two years, as many as six air
defence and anti-tank missile projects have
been built in India by the Defence Research and
Development Organisation (DRDO), even as
several new ones are under its consideration.
The collaboration with the foreign players will
lead to transfer of technology and funds.

113. Correct option: (a)
Explanation:
Option (a) is correct
Supplementary notes
 The demand of collegium system was fi rst
raised in 1990 when the Dinesh Goswami
Committee suggested the need for a selection
committee or a panel to appoint the CEC (at
that time ECI was a single-member body).
Based on this recommendation, 70th
Constitutional Amendment Bill, 1990 was
introduced in the Parliament which demanded
for the selection committee comprising of the
Presiding Offi cers of both Houses and Leader
of the Opposition in Lok Sabha.
However, due to lack of political will, it was
not passed and was withdrawn in 1993. Till
date, no such bill was formed.
The issue was debated in the Constituent
Assembly which finally left over to the
government to decide the appointments. The
constituent Assembly suggested that election
commission should be appointed after a
proposal of the same is supported by twothird majority of the parliament.
Global Practice: In the USA, South Africa,
Canada, etc. there is a mechanism for outside
consultation with expert body for making
suitable appointments.
Recently, the Supreme Court has agreed to
hear a public interest litigation seeking that
the chief election commissioner and election
commissioners be appointed by a threemember collegium.
 The collegium will comprise the Prime Minister,
the leader of opposition in Lok Sabha and
the Chief Justice of India. A bench comprising
Chief Justice S. A. Bobde and Justices B.R.
Gavai and Surya Kant took note of submissions
that the plea needed an urgent hearing.
Advocate Ashwini Upadhyay filed the PIL
seeking to ensure more autonomy for the chief
election commissioner’s offi ce and election
commissioners. The plea has also sought
an independent secretariat for the Election
Commission of India and that it should also
be given the power to make rules.
The Election Commission of India (ECI) is
an autonomous constitutional authority
responsible for administering Union and State
election processes in India.
 The body administers elections to the Lok Sabha,
Rajya Sabha, State Legislative Assemblies, and
the offi ces of the President and Vice President
in the country.

114. Correct option: (a)
Explanation:
Option (a) is correct
Supplementary notes
Basins and Categories
Sedimentary Basins of India
The existing 26 Sedimentary Basins have an
area of approximately 3.14 million Sq. Kms. The
sedimentary basins of the country have been
classifi ed into four categories as below:
Category-I
! Basins with established commercial
production. Cambay, Mumbai Offshore,
Rajasthan, Krishna Godavari, Cauvery, Assam
Shelf and Assam-Arakan Fold Belt
Category-II
! Basins with known accumulation of
hydrocarbons but no commercial production
achieved so far
! Kutch, Mahanadi-NEC (North East Coast)
Basin,Andaman-Nicobar, Kerala-KonkanLakshadweep Basin.
Category-III
! Basins having hydrocarbon shows that
are considered geologically prospective
Himalayan Foreland Basin, Ganga Basin,
Vindhyan basin, Saurashtra Basin, Kerela
Konkan Basin, Bengal Basin
Category-IV
! Basins having uncertain potential which may
be prospective by analogy with similar basins
in the world. Karewa basin, Spiti-Zanskar
basin, Satpura–South Rewa–Damodar basin,
Chhattisgarh Basin, Narmada basin, Deccan
Syneclise, Bhima-Kaladgi, Bastar Basin,
PranhitaGodavari basin, Cuddapah basin.

115. Correct option: (a)
Explanation:
4thstatement is incorrect. Despite being one
of the largest consumers of energy, India has a
low share of MNCs in the domestic market.
Supplementary notes
Key Problems of Oil and Gas Sector
Declining domestic crude production: Most
of the producing fi elds (in Cambay, AssamArakan and Mumbai Offshore) are maturing
or have already matured. Due to inadequate
new oil and gas discoveries and subsequent
development, India is witnessing a decline in
crude production.

116. Correct option: (c)
Explanation:
Statement 2 is incorrect: Quick Reaction
Surface to Air Missile (QRSAM) system developed
by Defence Research and Development
Organization (DRDO) was successfully fl ighttested from Integrated Test Range, Chandipur
off the Odisha coast at 1145 hrs on December
23, 2019.
Supplementary notes
Quick Reaction Surface to Air Missile
(QRSAM) system
 They are developed by Defence Research and
Development Organisation (DRDO).
 The missile has been fl ight-tested with full
confi guration in deployment mode intercepting
the target mid-air, meeting the mission
objectives.

117. Correct option: (d)
Explanation:
All the statements are correct: The Union
Cabinet chaired by Prime Minister has approved
to create the post of Chief of Defence Staff
in the rank of a four-star General with salary
and perquisites equivalent to a Service Chief.
The Chief of Defence Staff will also head the
Department of Military Affairs (DMA), to be
created within the Ministry of Defence and
function as its Secretary.
Supplementary notes
Chief of Defence Staff
 The Chief of Defence Staff, apart from being
the head of the Department of Military Affairs,
will also be the Permanent Chairman of the
Chiefs of Staff Committee.
He will act as the Principal Military Adviser to
Raksha Mantri on all tri-Services matters. The
three Chiefs will continue to advise RM on
matters exclusively concerning their respective
Services.
CDS will not exercise any military command,
including over the three Service Chiefs, so as
to be able to provide impartial advice to the
political leadership.
The following areas will be dealt by the
Department of Military Affairs headed by
CDS:
! The Armed Forces of the Union, namely, the
Army, the Navy and the Air Force.
! Integrated Headquarters of the Ministry of
Defence comprising Army Headquarters,
Naval Headquarters, Air Headquarters and
Defence Staff Headquarters.
! The Territorial Army.
! Works relating to the Army, the Navy and
the Air Force.
! Procurement exclusive to the Services except
capital acquisitions, as per prevalent rules
and procedures.
Apart from the above, the mandate of the
Department of Military Affairs will include
the following areas:
! Promoting jointness in procurement,
training and staffing for the Services
through joint planning and integration of
their requirements.
! Function as the Military Adviser to the
Nuclear Command Authority.
! Bring about jointness in operation, logistics,
transport, training, support services,
communications, repairs and maintenance,
etc of the three Services, within three years
of the fi rst CDS assuming offi ce.
! Ensure optimal utilisation of infrastructure
and rationalise it through jointness among
the services.

118. Correct option: (d)
Explanation:
All the statements are correct: The Union
Cabinet chaired by the Prime Minister has
given its approval for the implementation of
the Atal Bhujal Yojana (ATAL JAL), a Central
Sector Scheme with a total outlay of Rs.6000
crore to be implemented over a period of 5
years (2020-21 to 2024-25).
Supplementary notes
Ground water contributes to nearly 65% of
total irrigated area of the country and nearly
85% of the rural drinking water supply. The
limited ground water resources in the country
are under threat due to the increasing demands
of growing population, urbanization and
industrialization.
 Intensive, and unregulated ground water
pumping in many areas has caused rapid and
widespread decline in ground water levels as
well as reduction in the sustainability of ground
water abstraction structures.
 Incentive Component for incentivising
the States for achievements in improved
groundwater management practices namely,
data dissemination, preparation of water
security plans, implementation of management
interventions through convergence of ongoing
schemes, adopting demand side management
practices etc
ATAL JAL will result in:
Institutional strengthening for improving
ground water monitoring networks and capacity
building of stakeholders at different levels
which will enhance ground water data storage,
exchange, analysis and dissemination.
Improved and realistic water budgeting based
on an improved database and preparation
of community-led Water Security Plans at
Panchayat level
Implementation of Water Security Plans
through convergence of various ongoing/
new schemes of the Government of India and
State Governments to facilitate judicious and
effective utilization of funds for sustainable
ground water management.
 Effi cient use of available ground water resources
with emphasis on demand side measures
such as micro-irrigation, crop diversifi cation,
electricity feeder separation etc.
Impact:
 Source sustainability for Jal Jeevan Mission in
the project area with active participation of
local communities.
Will contribute towards the goal of doubling
the farmers’ income.
Will promote participatory ground water
management.
 Improved water use effi ciency on a mass scale
and improved cropping pattern;
Promotion of efficient and equitable use
of ground water resources and behavioural
change at the community level.

119. Correct option: (a)
Explanation:
Statement 3 is incorrect: The Union Cabinet
chaired by Prime Minister has approved a
transformational organisational restructuring
of the Indian Railways.
The reforms include:
! Unification of the existing eight Group
A services of the Railways into a Central
Service called Indian Railway Management
Service (/RMS)
! Re-organisation of Railway Board on
functional lines headed by CRB with
four Members and some Independent
Members

120. Correct option: (c)
Explanation:
Statement 1 and 2 are correct: Decennial
Population Census is being conducted in
India synchronously since 1872 without break.
Census 2021 will be 16th Census in the country
and 8th after independence.

121. Correct option: (c)
Explanation:
Statement 1 and 2 are correct: In a major step
towards making India a gas-based economy
and making CNG as the eco-friendly option for
long distance transport in the country, Minister
of Petroleum & Natural Gas and Steel, unveiled
India’s fi rst long distance CNG bus fi tted with
composite CNG cylinders, which can travel
around 1000 kms in a single fi ll.
 The project has been executed by Indraprastha
Gas Limited (IGL) and has been achieved
through pioneering design of Type IV Composite
Cylinders in buses, replacing traditional very
heavy Type-I Carbon Steel cylinders.
Supplementary notes

122. Correct option: (d)
Explanation:
All three Statements are correct: NITI
Aayog will launch the second edition of the
Sustainable Development Goals (SDG) India
Index, which documents the progress made
by India’s States and Union Territories towards
implementing the 2030 SDG targets, on 30
December 2019 at NITI Aayog, New Delhi.
The SDG India Index and Dashboard 2019–20
have been developed in collaboration with
the Ministry of Statistics and Programme
Implementation (MoSPI), the United Nations in
India, and the Global Green Growth Institute.
Supplementary notes

123. Correct option: (b)
Explanation:
Statement 2 and 3 are incorrect: The GGI
takes into consideration ten sectors: 1).
Agriculture and Allied Sectors, 2). Commerce &
Industries, 3). Human Resource Development,
4). Public Health, 5). Public Infrastructure &
Utilities, 6). Economic Governance, 7). Social
Welfare & Development, 8). Judicial & Public
Security, 9). Environment and 10). Citizen Centric Governance.